NR 450 Exam 3

Lakukan tugas rumah & ujian kamu dengan baik sekarang menggunakan Quizwiz!

A nurse is assessing a client who has suffered a nasal fracture. Which assessment should the nurse perform first? a. Facial pain b. Vital signs c. Bone displacement d. Airway patency

D A patent airway is the priority. The nurse first should make sure that the airway is patent and then should determine whether the client is in pain and whether bone displacement or blood loss has occurred.

A nurse cares for a client who is experiencing epistaxis. Which action should the nurse take first? a. Initiate Standard Precautions. b. Apply direct pressure. c. Sit the client upright. d. Loosely pack the nares with gauze.

ANS: A The nurse should implement Standard Precautions and don gloves prior to completing the other actions.

A nurse is caring for a client with a nonhealing arterial lower leg ulcer. What action by the nurse is best? a. Consult with the Wound Ostomy Care Nurse. b. Give pain medication prior to dressing changes. c. Maintain sterile technique for dressing changes. d. Prepare the client for eventual amputation.

ANS: A A nonhealing wound needs the expertise of the Wound Ostomy Care Nurse (or Wound Ostomy Continence Nurse). Premedicating prior to painful procedures and maintaining sterile technique are helpful, but if the wound is not healing, more needs to be done. The client may need an amputation, but other options need to be tried first.

A nurse is teaching a larger female client about alcohol intake and how it affects hypertension. The client asks if drinking two beers a night is an acceptable intake. What answer by the nurse is best? a. No, women should only have one beer a day as a general rule. b. No, you should not drink any alcohol with hypertension. c. Yes, since you are larger, you can have more alcohol. d. Yes, two beers per day is an acceptable amount of alcohol.

ANS: A Alcohol intake should be limited to two drinks a day for men and one drink a day for women. A drink is classified as one beer, 1.5 ounces of hard liquor, or 5 ounces of wine. Limited alcohol intake is acceptable with hypertension. The womans size does not matter.

A nurse is assessing a client with peripheral artery disease (PAD). The client states walking five blocks is possible without pain. What question asked next by the nurse will give the best information? a. Could you walk further than that a few months ago?b. Do you walk mostly uphill, downhill, or on flat surfaces? c. Have you ever considered swimming instead of walking? d. How much pain medication do you take each day?

ANS: A As PAD progresses, it takes less oxygen demand to cause pain. Needing to cut down on activity to be pain free indicates the clients disease is worsening. The other questions are useful, but not as important.

A nurse assesses a client with tachycardia. Which clinical manifestation requires immediate intervention by the nurse? a. Mid-sternal chest pain b. Increased urine output c. Mild orthostatic hypotension d. P wave touching the T wave

ANS: A Chest pain, possibly angina, indicates that tachycardia may be increasing the clients myocardial workload and oxygen demand to such an extent that normal oxygen delivery cannot keep pace. This results in myocardial hypoxia and pain. Increased urinary output and mild orthostatic hypotension are not life-threatening conditions and therefore do not require immediate intervention. The P wave touching the T wave indicates significant tachycardia and should be assessed to determine the underlying rhythm and cause; this is an important assessment but is not as critical as chest pain, which indicates cardiac cell death.

A nursing student is caring for a client who had a myocardial infarction. The student is confused because the client states nothing is wrong and yet listens attentively while the student provides education on lifestyle changes and healthy menu choices. What response by the faculty member is best? a. Continue to educate the client on possible healthy changes. b. Emphasize complications that can occur with noncompliance. c. Tell the client that denial is normal and will soon go away. d. You need to make sure the client understands this illness.

ANS: A Clients are often in denial after a coronary event. The client who seems to be in denial but is compliant with treatment may be using a healthy form of coping that allows time to process the event and start to use problem- focused coping. The student should not discourage this type of denial and coping, but rather continue providing education in a positive manner. Emphasizing complications may make the client defensive and more anxious. Telling the client that denial is normal is placing too much attention on the process. Forcing the client to verbalize understanding of the illness is also potentially threatening to the client.

A client has peripheral arterial disease (PAD). What statement by the client indicates misunderstanding about self-management activities? a. I can use a heating pad on my legs if its set on low. b. I should not cross my legs when sitting or lying down. c. I will go out and buy some warm, heavy socks to wear. d. Its going to be really hard but I will stop smoking.

ANS: A Clients with PAD should never use heating pads as skin sensitivity is diminished and burns can result. The other statements show good understanding of self-management.

A client had an inferior wall myocardial infarction (MI). The nurse notes the clients cardiac rhythm as shown below: What action by the nurse is most important? a. Assess the clients blood pressure and level of consciousness. b. Call the health care provider or the Rapid Response Team. c. Obtain a permit for an emergency temporary pacemaker insertion. d. Prepare to administer antidysrhythmic medication.

ANS: A Clients with an inferior wall MI often have bradycardia and blocks that lead to decreased perfusion, as seen in this ECG strip showing sinus bradycardia. The nurse should first assess the clients hemodynamic status, including vital signs and level of consciousness. The client may or may not need the Rapid Response Team, a temporary pacemaker, or medication; there is no indication of this in the question.

While assessing a client on a cardiac unit, a nurse identifies the presence of an S3 gallop. Which action should the nurse take next? a. Assess for symptoms of left-sided heart failure. b. Document this as a normal finding. c. Call the health care provider immediately. d. Transfer the client to the intensive care unit.

ANS: A The presence of an S3 gallop is an early diastolic filling sound indicative of increasing left ventricular pressure and left ventricular failure. The other actions are not warranted.

A nurse cares for an older adult client with heart failure. The client states, I dont know what to do. I dont want to be a burden to my daughter, but I cant do it alone. Maybe I should die. How should the nurse respond? a. Would you like to talk more about this? b. You are lucky to have such a devoted daughter. c. It is normal to feel as though you are a burden. d. Would you like to meet with the chaplain?

ANS: A Depression can occur in clients with heart failure, especially older adults. Having the client talk about his or her feelings will help the nurse focus on the actual problem. Open-ended statements allow the client to respond safely and honestly. The other options minimize the clients concerns and do not allow the nurse to obtain more information to provide client-centered care.

A client is on a dopamine infusion via a peripheral line. What action by the nurse takes priority for safety? a. Assess the IV site hourly. b. Monitor the pedal pulses. c. Monitor the clients vital signs. d. Obtain consent for a central line.

ANS: A Dopamine should be infused through a central line to prevent extravasation and necrosis of tissue. If it needs to be run peripherally, the nurse assesses the site hourly for problems. When the client is getting the central line, ensuring informed consent is on the chart is a priority. But at this point, the client has only a peripheral line, so caution must be taken to preserve the integrity of the clients integumentary system. Monitoring pedal pulses and vital signs give indications as to how well the drug is working.

While assessing a client who has facial trauma, the nurse auscultates stridor. The client is anxious and restless. Which action should the nurse take first? a. Contact the provider and prepare for intubation. b. Administer prescribed albuterol nebulizer therapy. c. Place the client in high-Fowlers position. d. Ask the client to perform deep-breathing exercises.

ANS: A Facial and neck tissue edema can occur in clients with facial trauma. Airway patency is the highest priority. Clients who experience stridor and hypoxia, manifested by anxiety and restlessness, should be immediately intubated to ensure airway patency. Albuterol decreases bronchi and bronchiole inflammation, not facial and neck edema. Although putting the client in high-Fowlers position and asking the client to perform breathing exercises may temporarily improve the clients comfort, these actions will not decrease the underlying problem or improve airway patency.

The nurse is caring for four hypertensive clients. Which druglaboratory value combination should the nurse report immediately to the health care provider? a. Furosemide (Lasix)/potassium: 2.1 mEq/L b. Hydrochlorothiazide (Hydrodiuril)/potassium: 4.2 mEq/L c. Spironolactone (Aldactone)/potassium: 5.1 mEq/L d. Torsemide (Demadex)/sodium: 142 mEq/L

ANS: A Lasix is a loop diuretic and can cause hypokalemia. A potassium level of 2.1 mEq/L is quite low and should be reported immediately. Spironolactone is a potassium-sparing diuretic that can cause hyperkalemia. A potassium level of 5.1 mEq/L is on the high side, but it is not as critical as the low potassium with furosemide. The other two laboratory values are normal.

A client is taking warfarin (Coumadin) and asks the nurse if taking St. Johns wort is acceptable. What response by the nurse is best? a. No, it may interfere with the warfarin. b. There isnt any information about that. c. Why would you want to take that? d. Yes, it is a good supplement for you.

ANS: A Many foods and drugs interfere with warfarin, St. Johns wort being one of them. The nurse should advise the client against taking it. The other answers are not accurate.

A nurse cares for a client after radiation therapy for lung cancer. The client reports a sore throat. Which action should the nurse take first? a. Ask the client to gargle with mouthwash containing lidocaine. b. Administer prescribed intravenous pain medications. c. Explain that soreness is normal and will improve in a couple days. d. Assess the clients neck for redness and swelling.

ANS: A Mouthwashes and throat sprays containing a local anesthetic agent such as lidocaine or diphenhydramine can provide relief from a sore throat after radiation therapy. Intravenous pain medications may be used if local anesthetics are unsuccessful. The nurse should explain to the client that this is normal and assess the clients neck, but these options do not decrease the clients discomfort.

A client is receiving norepinephrine (Levophed) for shock. What assessment finding best indicates a therapeutic effect from this drug? a. Alert and oriented, answering questions b. Client denial of chest pain or chest pressure c. IV site without redness or swelling d. Urine output of 30 mL/hr for 2 hours

ANS: A Normal cognitive function is a good indicator that the client is receiving the benefits of norepinephrine. The brain is very sensitive to changes in oxygenation and perfusion. Norepinephrine can cause chest pain as an adverse reaction, so the absence of chest pain does not indicate therapeutic effect. The IV site is normal. The urine output is normal, but only minimally so.

A nurse teaches a client who experiences occasional premature atrial contractions (PACs) accompanied by palpitations that resolve spontaneously without treatment. Which statement should the nurse include in this clients teaching? a. Minimize or abstain from caffeine. b. Lie on your side until the attack subsides. c. Use your oxygen when you experience PACs. d. Take amiodarone (Cordarone) daily to prevent PACs.

ANS: A PACs usually have no hemodynamic consequences. For a client experiencing infrequent PACs, the nurse should explore possible lifestyle causes, such as excessive caffeine intake and stress. Lying on the side will not prevent or resolve PACs. Oxygen is not necessary. Although medications may be needed to control symptomatic dysrhythmias, for infrequent PACs, the client first should try lifestyle changes to control them.

A nurse auscultates a clients lung fields. Which action should the nurse take based on the lung sounds? (Click the media button to hear the audio clip.) a. Assess for airway obstruction. b. Initiate oxygen therapy. c. Assess vital signs. d. Elevate the clients head.

ANS: A Stridor is the sound heard, and it indicates severe airway constriction. The nurse must administer a bronchodilator to get air into the lungs. Administering oxygen, assessing vital signs, and elevating the clients head will not help until the clients airways are open.

A nurse assists with the cardioversion of a client experiencing acute atrial fibrillation. Which action should the nurse take prior to the initiation of cardioversion? a. Administer intravenous adenosine. b. Turn off oxygen therapy. c. Ensure a tongue blade is available. d. Position the client on the left side.

ANS: B For safety during cardioversion, the nurse should turn off any oxygen therapy to prevent fire. The other interventions are not appropriate for a cardioversion. The client should be placed in a supine position.

A nurse assesses a client who is prescribed fluticasone (Flovent) and notes oral lesions. Which action should the nurse take? a. Encourage oral rinsing after fluticasone administration. b. Obtain an oral specimen for culture and sensitivity. c. Start the client on a broad-spectrum antibiotic. d. Document the finding as a known side effect.

ANS: A The drug reduces local immunity and increases the risk for local infection, especially Candida albicans. Rinsing the mouth after using the inhaler will decrease the risk for developing this infection. Use of mouthwash and broad-spectrum antibiotics is not warranted in this situation. The nurse should document the finding, but the best action to take is to have the client start rinsing his or her mouth after using fluticasone. An oral specimen for culture and sensitivity will not provide information necessary to care for this client.

A client has been diagnosed with a deep vein thrombosis and is to be discharged on warfarin (Coumadin). The client is adamant about refusing the drug because its dangerous. What action by the nurse is best? a. Assess the reason behind the clients fear. b. Remind the client about laboratory monitoring. c. Tell the client drugs are safer today than before. d. Warn the client about consequences of noncompliance.

ANS: A The first step is to assess the reason behind the clients fear, which may be related to the experience of someone the client knows who took warfarin. If the nurse cannot address the specific rationale, teaching will likely be unsuccessful. Laboratory monitoring once every few weeks may not make the client perceive the drug to be safe. General statements like drugs are safer today do not address the root cause of the problem. Warning the client about possible consequences of not taking the drug is not therapeutic and is likely to lead to an adversarial relationship.

A nurse prepares to discharge a client with cardiac dysrhythmia who is prescribed home health care services. Which priority information should be communicated to the home health nurse upon discharge? a. Medication reconciliation b. Immunization history c. Religious beliefs d. Nutrition preferences

ANS: A The home health nurse needs to know current medications the client is taking to ensure assessment, evaluation, and further education related to these medications. The other information will not assist the nurse to develop a plan of care for the client.

A client in shock is apprehensive and slightly confused. What action by the nurse is best? a. Offer to remain with the client for awhile. b. Prepare to administer antianxiety medication. c. Raise all four siderails on the clients bed. d. Tell the client everything possible is being done.

ANS: A The nurses presence will be best to reassure this client. Antianxiety medication is not warranted as this will lower the clients blood pressure. Using all four siderails on a hospital bed is considered a restraint in most facilities, although the nurse should ensure the clients safety. Telling a confused client that everything is being done is not the most helpful response.

A client has been brought to the emergency department after being shot multiple times. What action should the nurse perform first? a. Apply personal protective equipment. b. Notify local law enforcement officials. c. Obtain universal donor blood. d. Prepare the client for emergency surgery.

ANS: A The nurses priority is to care for the client. Since the client has gunshot wounds and is bleeding, the nurse applies personal protective equipment (i.e., gloves) prior to care. This takes priority over calling law enforcement. Requesting blood bank products can be delegated. The nurse may or may not have to prepare the client for emergency surgery.

The nurse instructs a client on the steps needed to obtain a peak expiratory flow rate. In which order should these steps occur? 1. Take as deep a breath as possible. 2. Stand up (unless you have a physical disability). 3. Place the meter in your mouth, and close your lips around the mouthpiece. 4. Make sure the device reads zero or is at base level. 5. Blow out as hard and as fast as possible for 1 to 2 seconds .6. Write down the value obtained. 7. Repeat the process two additional times, and record the highest number in your chart. a. 4, 2, 1, 3, 5, 6, 7 b. 3, 4, 1, 2, 5, 7, 6 c. 2, 1, 3, 4, 5, 6, 7 d. 1, 3, 2, 5, 6, 7, 4

ANS: A The proper order for obtaining a peak expiratory flow rate is as follows. Make sure the device reads zero or is at base level. The client should stand up (unless he or she has a physical disability). The client should take as deep a breath as possible, place the meter in the mouth, and close the lips around the mouthpiece. The client should blow out as hard and as fast as possible for 1 to 2 seconds. The value obtained should be written down. The process should be repeated two more times, and the highest of the three numbers should be recorded in the clients chart.

A nurse is working with a client who takes atorvastatin (Lipitor). The clients recent laboratory results include a blood urea nitrogen (BUN) of 33 mg/dL and creatinine of 2.8 mg/dL. What action by the nurse is best? a. Ask if the client eats grapefruit. b. Assess the client for dehydration. c. Facilitate admission to the hospital. d. Obtain a random urinalysis.

ANS: A There is a drug-food interaction between statins and grapefruit that can lead to acute kidney failure. This client has elevated renal laboratory results, indicating some degree of kidney involvement. The nurse should assess if the client eats grapefruit or drinks grapefruit juice. Dehydration can cause the BUN to be elevated, but the elevation in creatinine is more specific for a kidney injury. The client does not necessarily need to be admitted. A urinalysis may or may not be ordered.

The nurse is assessing a client on admission to the hospital. The clients leg appears as shown below: What action by the nurse is best? a. Assess the clients ankle-brachial index. b. Elevate the clients leg above the heart. c. Obtain an ice pack to provide comfort. d. Prepare to teach about heparin sodium.

ANS: A This client has dependent rubor, a classic finding in peripheral arterial disease. The nurse should measure the clients ankle-brachial index. Elevating the leg above the heart will further impede arterial blood flow. Ice will cause vasoconstriction, also impeding circulation and perhaps causing tissue injury. Heparin sodium is not the drug of choice for this condition.

A nurse supervises an unlicensed assistive personnel (UAP) applying electrocardiographic monitoring. Which statement should the nurse provide to the UAP related to this procedure? a. Clean the skin and clip hairs if needed. b. Add gel to the electrodes prior to applying them. c. Place the electrodes on the posterior chest. d. Turn off oxygen prior to monitoring the client.

ANS: A To ensure the best signal transmission, the skin should be clean and hairs clipped. Electrodes should be placed on the anterior chest, and no additional gel is needed. Oxygen has no impact on electrocardiographic monitoring.

A nurse cares for a client who is on a cardiac monitor. The monitor displayed the rhythm shown below: Which action should the nurse take first? a. Assess airway, breathing, and level of consciousness. b. Administer an amiodarone bolus followed by a drip. c. Cardiovert the client with a biphasic defibrillator. d. Begin cardiopulmonary resuscitation (CPR).

ANS: A Ventricular tachycardia occurs with repetitive firing of an irritable ventricular ectopic focus, usually at a rate of 140 to 180 beats/min or more. Ventricular tachycardia is a lethal dysrhythmia. The nurse should first assess if the client is alert and breathing. Then the nurse should call a Code Blue and begin CPR. If this client is pulseless, the treatment of choice is defibrillation. Amiodarone is the antidysrhythmic of choice, but it is not the first action.

A nurse auscultates a clients lung fields. Which pathophysiologic process should the nurse associate with this breath sound?(Click the media button to hear the audio clip.) a. Inflammation of the pleura b. Constriction of the bronchioles c. Upper airway obstruction d. Pulmonary vascular edema

ANS: A A pleural friction rub can be heard when the pleura is inflamed and rubbing against the lung wall. The other pathophysiologic processes would not cause a pleural friction rub. Constriction of the bronchioles may be heard as a wheeze, upper airway obstruction may be heard as stridor, and pulmonary vascular edema may be heard as crackles.

A client has been diagnosed with a very large pulmonary embolism (PE) and has a dropping blood pressure. What medication should the nurse anticipate the client will need as the priority? A. Alteplase (Activase) B. Enoxaparin (Lovenox) C. Unfractionated heparin D. Warfarin sodium (Coumadin)

ANS: A Activase is a clot-busting agent indicated in large PEs in the setting of hemodynamic instability. The nurse knows this drug is the priority, although heparin may be started initially. Enoxaparin and warfarin are not indicated in this setting.

A nurse is interested in providing community education and screening on hypertension. In order to reach a priority population, to what target audience should the nurse provide this service? a. African-American churches b. Asian-American groceries c. High school sports camps d. Womens health clinics

ANS: A African Americans in the United States have one of the highest rates of hypertension in the world. The nurse has the potential to reach this priority population by providing services at African-American churches. Although hypertension education and screening are important for all groups, African Americans are the priority population for this intervention.

A client in the emergency department has several broken ribs. What care measure will best promote comfort? a. Allowing the client to choose the position in bed b. Humidifying the supplemental oxygen c. Offering frequent, small drinks of water d.Providing warmed blankets

ANS: A Allow the client with respiratory problems to assume a position of comfort if it does not interfere with care. Often the client will choose a more upright position, which also improves oxygenation. The other options are less effective comfort measures.

A nurse is caring for a client who is intubated and has an intra-aortic balloon pump. The client is restless and agitated. What action should the nurse perform first for comfort? a. Allow family members to remain at the bedside. b. Ask the family if the client would like a fan in the room. c. Keep the television tuned to the clients favorite channel. d. Speak loudly to the client in case of hearing problems.

ANS: A Allowing the family to remain at the bedside can help calm the client with familiar voices (and faces if the client wakes up). A fan might be helpful but may also spread germs through air movement. The TV should not be kept on all the time to allow for rest. Speaking loudly may agitate the client more.

A nurse assesses clients on a cardiac unit. Which client should the nurse identify as being at greatest risk for the development of left-sided heart failure? a. A 36-year-old woman with aortic stenosis b. A 42-year-old man with pulmonary hypertension c. A 59-year-old woman who smokes cigarettes daily d. A 70-year-old man who had a cerebral vascular accident

ANS: A Although most people with heart failure will have failure that progresses from left to right, it is possible to have left-sided failure alone for a short period. It is also possible to have heart failure that progresses from right to left. Causes of left ventricular failure include mitral or aortic valve disease, coronary artery disease, and hypertension. Pulmonary hypertension and chronic cigarette smoking are risk factors for right ventricular failure. A cerebral vascular accident does not increase the risk of heart failure.

A nurse is teaching a client with heart failure who has been prescribed enalapril (Vasotec). Which statement should the nurse include in this clients teaching? a. Avoid using salt substitutes. b. Take your medication with food. c. Avoid using aspirin-containing products. d. Check your pulse daily.

ANS: A Angiotensin-converting enzyme (ACE) inhibitors such as enalapril inhibit the excretion of potassium. Hyperkalemia can be a life-threatening side effect, and clients should be taught to limit potassium intake. Salt substitutes are composed of potassium chloride. ACE inhibitors do not need to be taken with food and have no impact on the clients pulse rate. Aspirin is often prescribed in conjunction with ACE inhibitors and is not contraindicated.

A nurse admits a client who is experiencing an exacerbation of heart failure. Which action should the nurse take first? a. Assess the clients respiratory status. b. Draw blood to assess the clients serum electrolytes. c. Administer intravenous furosemide (Lasix). d. Ask the client about current medications.

ANS: A Assessment of respiratory and oxygenation status is the priority nursing intervention for the prevention of complications. Monitoring electrolytes, administering diuretics, and asking about current medications are important but do not take priority over assessing respiratory status.

An emergency room nurse assesses a client who was rescued from a home fire. The client suddenly develops a loud, brassy cough. Which action should the nurse take first? a. Apply oxygen and continuous pulse oximetry. b. Provide small quantities of ice chips and sips of water. c. Request a prescription for an antitussive medication. d. Ask the respiratory therapist to provide humidified air.

ANS: A Brassy cough and wheezing are some of the signs seen with inhalation injury. The first action by the nurse is to give the client oxygen. Clients with possible inhalation injury also need continuous pulse oximetry. Ice chips and humidified room air will not help the problem, and antitussives are not warranted.

A nurse assesses a client with tachycardia. Which clinical manifestation requires immediate intervention by the nurse? a. Midsternal chest pain b. Increased urine output c. Mild orthostatic hypotension d. P wave touching the T wave

ANS: A Chest pain, possibly angina, indicates that tachycardia may be increasing the client's myocardial workload and oxygen demand to such an extent that normal oxygen delivery cannot keep pace. This results in myocardial hypoxia and pain. Increased urinary output and mild orthostatic hypotension are not life-threatening conditions and therefore do not require immediate intervention. The P wave touching the T wave indicates significant tachycardia and would be assessed to determine the underlying rhythm and cause; this is an important assessment but is not as critical as chest pain, which indicates cardiac cell death.

A client is in the clinic a month after having a myocardial infarction. The client reports sleeping well since moving into the guest bedroom. What response by the nurse is best? a. Do you have any concerns about sexuality? b. Im glad to hear you are sleeping well now. c. Sleep near your spouse in case of emergency. d. Why would you move into the guest room?

ANS: A Concerns about resuming sexual activity are common after cardiac events. The nurse should gently inquire if this is the issue. While it is good that the client is sleeping well, the nurse should investigate the reason for the move. The other two responses are likely to cause the client to be defensive.

A nurse cares for a client with right-sided heart failure. The client asks, Why do I need to weigh myself every day? How should the nurse respond? a. Weight is the best indication that you are gaining or losing fluid. b. Daily weights will help us make sure that youre eating properly. c. The hospital requires that all inpatients be weighed daily. d. You need to lose weight to decrease the incidence of heart failure.

ANS: A Daily weights are needed to document fluid retention or fluid loss. One liter of fluid equals 2.2 pounds. The other responses do not address the importance of monitoring fluid retention or loss.

A nurse cares for a client with burn injuries. Which intervention should the nurse implement to appropriately reduce the clients pain? a. Administer the prescribed intravenous morphine sulfate. b. Apply ice to skin around the burn wound for 20 minutes. c. Administer prescribed intramuscular ketorolac (Toradol). d. Decrease tactile stimulation near the burn injuries.

ANS: A Drug therapy for pain management requires opioid and nonopioid analgesics. The IV route is used because of problems with absorption from the muscle and the stomach. For the client to avoid shivering, the room must be kept warm, and ice should not be used. Ice would decrease blood flow to the area. Tactile stimulation can be used for pain management.

The nurse assesses a client who has a severe burn injury. Which statement indicates the client understands the psychosocial impact of a severe burn injury? a. It is normal to feel some depression. b. I will go back to work immediately. c. I will not feel anger about my situation. d. Once I get home, things will be normal.

ANS: A During the recovery period, and for some time after discharge from the hospital, clients with severe burn injuries are likely to have psychological problems that require intervention. Depression is one of these problems. Grief, loss, anxiety, anger, fear, and guilt are all normal feelings that can occur. Clients need to know that problems of physical care and psychological stresses may be overwhelming.

An emergency room nurse obtains the health history of a client. Which statement by the client should alert the nurse to the occurrence of heart failure? a. I get short of breath when I climb stairs. b. I see halos floating around my head. c. I have trouble remembering things. d. I have lost weight over the past month.

ANS: A Dyspnea on exertion is an early manifestation of heart failure

A nurse administers topical gentamicin sulfate (Garamycin) to a clients burn injury. Which laboratory value should the nurse monitor while the client is prescribed this therapy? a. Creatinine b. Red blood cells c. Sodium d. Magnesium

ANS: A Gentamicin is nephrotoxic, and sufficient amounts can be absorbed through burn wounds to affect kidney function. Any client receiving gentamicin by any route should have kidney function monitored. Topical gentamicin will not affect the red blood cell count or the sodium or magnesium levels.

A client had a femoropopliteal bypass graft with a synthetic graft. What action by the nurse is most important to prevent wound infection? a. Appropriate hand hygiene before giving care b. Assessing the clients temperature every 4 hours c. Clean technique when changing dressings d. Monitoring the clients daily white blood cell count

ANS: A Hand hygiene is the best way to prevent infections in hospitalized clients. Dressing changes should be done with sterile technique. Assessing vital signs and white blood cell count will not prevent infection.

A client is in shock and the nurse prepares to administer insulin for a blood glucose reading of 208 mg/dL. The spouse asks why the client needs insulin as the client is not a diabetic. What response by the nurse is best? a. High glucose is common in shock and needs to be treated. b. Some of the medications we are giving are to raise blood sugar. c. The IV solution has lots of glucose, which raises blood sugar. d. The stress of this illness has made your spouse a diabetic.

ANS: A High glucose readings are common in shock, and best outcomes are the result of treating them and maintaining glucose readings in the normal range. Medications and IV solutions may raise blood glucose levels, but this is not the most accurate answer. The stress of the illness has not made the client diabetic.

A nurse delegates hydrotherapy to an unlicensed assistive personnel (UAP). Which statement should the nurse include when delegating this activity? a. Keep the water temperature constant when showering the client. b. Assess the wound beds during the hydrotherapy treatment. c. Apply a topical enzyme agent after bathing the client. d. Use sterile saline to irrigate and clean the clients wounds.

ANS: A Hydrotherapy is performed by showering the client on a special shower table. The UAP should keep the water temperature constant. This process allows the nurse to assess the wound beds, but a UAP cannot complete this act. Topical enzyme agents are not part of hydrotherapy. The irrigation does not need to be done with sterile saline.

A nurse assesses a client who is recovering from a myocardial infarction. The clients pulmonary artery pressure reading is 25/12 mm Hg. Which action should the nurse take first? a. Compare the results with previous pulmonary artery pressure readings. b. Increase the intravenous fluid rate because these readings are low. c. Immediately notify the health care provider of the elevated pressures. d. Document the finding in the clients chart as the only action.

ANS: A Normal pulmonary artery pressures range from 15 to 26 mm Hg for systolic and from 5 to 15 mm Hg for diastolic. Although this clients readings are within normal limits, the nurse needs to assess any trends that may indicate a need for medical treatment to prevent complications. There is no need to increase intravenous fluids or notify the provider.

An older adult is on cardiac monitoring after a myocardial infarction. The client shows frequent dysrhythmias. What action by the nurse is most appropriate? a. Assess for any hemodynamic effects of the rhythm. b. Prepare to administer antidysrhythmic medication. c. Notify the provider or call the Rapid Response Team. d. Turn the alarms off on the cardiac monitor.

ANS: A Older clients may have dysrhythmias due to age-related changes in the cardiac conduction system. They may have no significant hemodynamic effects from these changes. The nurse should first assess for the effects of the dysrhythmia before proceeding further. The alarms on a cardiac monitor should never be shut off. The other two actions may or may not be needed.

A nurse teaches a client who experiences occasional premature atrial contractions (PACs) accompanied by palpitations that resolve spontaneously without treatment. Which statement would the nurse include in this client's teaching? a. "Minimize or abstain from caffeine." b. "Lie on your side until the attack subsides." c. "Use your oxygen when you experience PACs." d. "Take amiodarone daily to prevent PACs."

ANS: A PACs usually have no hemodynamic consequences. For a client experiencing infrequent PACs, the nurse would explore possible lifestyle causes, such as excessive caffeine intake and stress. Lying on the side will not prevent or resolve PACs. Oxygen is not necessary. Although medications may be needed to control symptomatic dysrhythmias, for infrequent PACs, the client first would try lifestyle changes to control them.

A client appears dyspneic, but the oxygen saturation is 97%. What action by the nurse is best? A. Assess for other manifestations of hypoxia. B. Change the sensor on the pulse oximeter. C. Obtain a new oximeter from central supply. D. Tell the client to take slow, deep breaths.

ANS: A Pulse oximetry is not always the most accurate assessment tool for hypoxia as many factors can interfere, producing normal or near-normal readings in the setting of hypoxia. The nurse should conduct a more thorough assessment. The other actions are not appropriate for a hypoxic client.

A nurse cares for a client with a 40-year smoking history who is experiencing distended neck veins and dependent edema. Which physiologic process should the nurse correlate with this clients history and clinical manifestations? a. Increased pulmonary pressure creating a higher workload on the right side of the heart b. Exposure to irritants resulting in increased inflammation of the bronchi and bronchioles c. Increased number and size of mucus glands producing large amounts of thick mucus d. Left ventricular hypertrophy creating a decrease in cardiac output

ANS: A Smoking increases pulmonary hypertension, resulting in cor pulmonale, or right-sided heart failure. Increased pressures in the lungs make it more difficult for blood to flow through the lungs. Blood backs up into the right side of the heart and then into the peripheral venous system, creating distended neck veins and dependent edema. Inflammation in bronchi and bronchioles creates an airway obstruction which manifests as wheezes. Thick mucus in the lungs has no impact on distended neck veins and edema. Left ventricular hypertrophy is associated with left heart failure and is not caused by a 40-year smoking history.

A nurse assesses a client who has aortic regurgitation. In which location in the illustration shown below should the nurse auscultate to best hear a cardiac murmur related to aortic regurgitation? a. Location A b. Location B c. Location C d. Location D

ANS: A The aortic valve is auscultated in the second intercostal space just to the right of the sternum.

A nurse cares for a client with infective endocarditis. Which infection control precautions should the nurse use? a. Standard Precautions b. Bleeding precautions c. Reverse isolation d. Contact isolation

ANS: A The client with infective endocarditis does not pose any specific threat of transmitting the causative organism. Standard Precautions should be used. Bleeding precautions or reverse or contact isolation is not necessary.

A nurse assesses a client who has a nasal fracture. The client reports constant nasal drainage, a headache, and difficulty with vision. Which action should the nurse take next? a. Collect the nasal drainage on a piece of filter paper. b. Encourage the client to blow his or her nose. c. Perform a test focused on a neurologic examination. d. Palpate the nose, face, and neck.

ANS: A The client with nasal drainage after facial trauma could have a skull fracture that has resulted in leakage of cerebrospinal fluid (CSF). CSF can be differentiated from regular drainage by the fact that it forms a halo when dripped on filter paper. The other actions would be appropriate but are not as high a priority as assessing for CSF. A CSF leak would increase the clients risk for infection.

A nurse prepares to discharge a client with a cardiac dysrhythmia who is prescribed home health care services. Which priority information would be communicated to the home health nurse upon discharge? a. Medication orders for home b. Immunization history c. Religious beliefs d. Nutrition preferences

ANS: A The home health nurse needs to know current medications the client is taking to ensure assessment, evaluation, and further education related to these medications. The other information might be used to plan care, but not as the priority.

A nurse is caring for a client on mechanical ventilation and finds the client agitated and thrashing about. What action by the nurse is most appropriate? A. Assess the cause of the agitation. B. Reassure the client that he or she is safe. C. Restrain the clients hands. D. Sedate the client immediately.

ANS: A The nurse needs to determine the cause of the agitation. The inability to communicate often makes clients anxious, even to the point of panic. Pain and confusion can also cause agitation. Once the nurse determines the cause of the agitation, he or she can implement measures to relieve the underlying cause. Reassurance is also important but may not address the etiology of the agitation. Restraints and more sedation may be necessary, but not as a first step.

A client with a history of heart failure and hypertension is in the clinic for a follow-up visit. The client is on lisinopril (Prinivil) and warfarin (Coumadin). The client reports new-onset cough. What action by the nurse is most appropriate? a. Assess the clients lung sounds and oxygenation. b. Instruct the client on another antihypertensive. c. Obtain a set of vital signs and document them. d. Remind the client that cough is a side effect of Prinivil.

ANS: A This client could be having an exacerbation of heart failure or be experiencing a side effect of lisinopril (and other angiotensin-converting enzyme inhibitors). The nurse should assess the clients lung sounds and other signs of oxygenation first. The client may or may not need to switch antihypertensive medications. Vital signs and documentation are important, but the nurse should assess the respiratory system first. If the cough turns out to be a side effect, reminding the client is appropriate, but then more action needs to be taken.

The nurse gets the hand-off report on four clients. Which client should the nurse assess first? a. Client with a blood pressure change of 128/74 to 110/88 mm Hg b. Client with oxygen saturation unchanged at 94% c. Client with a pulse change of 100 to 88 beats/min d. Client with urine output of 40 mL/hr for the last 2 hours

ANS: A This client has a falling systolic blood pressure, rising diastolic blood pressure, and narrowing pulse pressure, all of which may be indications of the progressive stage of shock. The nurse should assess this client first. The client with the unchanged oxygen saturation is stable at this point. Although the client with a change in pulse has a slower rate, it is not an indicator of shock since the pulse is still within the normal range; it may indicate the clients pain or anxiety has been relieved, or he or she is sleeping or relaxing. A urine output of 40 mL/hr is only slightly above the normal range, which is 30 mL/hr.

A nurse supervises an assistive personnel (AP) applying electrocardiographic monitoring. Which statement would the nurse provide to the AP related to this procedure? a. "Clean the skin and clip hairs if needed." b. "Add gel to the electrodes prior to applying them." c. "Place the electrodes on the posterior chest." d. "Turn off oxygen prior to monitoring the client."

ANS: A To ensure the best signal transmission, the skin would be clean and hairs clipped. Electrodes would be placed on the anterior chest, and no additional gel is needed. Oxygen has no impact on electrocardiographic monitoring.

A nurse assesses a client who had a myocardial infarction and is hypotensive. Which additional assessment finding should the nurse expect? a. Heart rate of 120 beats/min b. Cool, clammy skin c. Oxygen saturation of 90% d. Respiratory rate of 8 breaths/min

ANS: A When a client experiences hypotension, baroreceptors in the aortic arch sense a pressure decrease in the vessels. The parasympathetic system responds by lessening the inhibitory effect on the sinoatrial node. This results in an increase in heart rate and respiratory rate. This tachycardia is an early response and is seen even when blood pressure is not critically low. An increased heart rate and respiratory rate will compensate for the low blood pressure and maintain oxygen saturations and perfusion. The client may not be able to compensate for long, and decreased oxygenation and cool, clammy skin will occur later.

A nurse teaches a client with heart failure about energy conservation. Which statement should the nurse include in this clients teaching? a. Walk until you become short of breath, and then walk back home. b. Gather everything you need for a chore before you begin. c. Pull rather than push or carry items heavier than 5 pounds. d. Take a walk after dinner every day to build up your strength.

ANS: B A client who has heart failure should be taught to conserve energy. Gathering all supplies needed for a chore at one time decreases the amount of energy needed. The client should not walk until becoming short of breath because he or she may not make it back home. Pushing a cart takes less energy than pulling or lifting. Although walking after dinner may help the client, the nurse should teach the client to complete activities when he or she has the most energy. This is usually in the morning.

A nurse is caring for several clients at risk for shock. Which laboratory value requires the nurse to communicate with the health care provider? a. Creatinine: 0.9 mg/dL b. Lactate: 6 mmol/L c. Sodium: 150 mEq/L d. White blood cell count: 11,000/mm3

ANS: B A lactate level of 6 mmol/L is high and is indicative of possible shock. A creatinine level of 0.9 mg/dL is normal. A sodium level of 150 mEq/L is high, but that is not related directly to shock. A white blood cell count of 11,000/mm3 is slightly high but is not as critical as the lactate level.

A client presents to the emergency department with a severely lacerated artery. What is the priority action for the nurse? a. Administer oxygen via non-rebreather mask. b. Ensure the client has a patent airway. c. Prepare to assist with suturing the artery. d. Start two large-bore IVs with normal saline.

ANS: B Airway always takes priority, followed by breathing and circulation. The nurse ensures the client has a patent airway prior to providing any other care measures.

A client arrives in the emergency department after being in a car crash with fatalities. The client has a nearly amputated leg that is bleeding profusely. What action by the nurse takes priority? a. Apply direct pressure to the bleeding. b. Ensure the client has a patent airway. c. Obtain consent for emergency surgery. d. Start two large-bore IV catheters.

ANS: B Airway is the priority, followed by breathing and circulation (IVs and direct pressure). Obtaining consent is done by the physician.

A client has hypertension and high risk factors for cardiovascular disease. The client is overwhelmed with the recommended lifestyle changes. What action by the nurse is best? a. Assess the clients support system. b. Assist in finding one change the client can control. c. Determine what stressors the client faces in daily life. d. Inquire about delegating some of the clients obligations.

ANS: B All options are appropriate when assessing stress and responses to stress. However, this client feels overwhelmed by the suggested lifestyle changes. Instead of looking at all the needed changes, the nurse should assist the client in choosing one the client feels optimistic about controlling. Once the client has mastered that change, he or she can move forward with another change. Determining support systems, daily stressors, and delegation opportunities does not directly impact the clients feelings of control.

A client is being discharged home after a large myocardial infarction and subsequent coronary artery bypass grafting surgery. The clients sternal wound has not yet healed. What statement by the client most indicates a higher risk of developing sepsis after discharge? a. All my friends and neighbors are planning a party for me. b. I hope I can get my water turned back on when I get home. c. I am going to have my daughter scoop the cat litter box. d. My grandkids are so excited to have me coming home!

ANS: B All these statements indicate a potential for leading to infection once the client gets back home. A large party might include individuals who are themselves ill and contagious. Having litter boxes in the home can expose the client to microbes that can lead to infection. Small children often have upper respiratory infections and poor hand hygiene that spread germs. However, the most worrisome statement is the lack of running water for handwashing and general hygiene and cleaning purposes.

A nurse wants to provide community service that helps meet the goals of Healthy People 2020 (HP2020) related to cardiovascular disease and stroke. What activity would best meet this goal? a. Teach high school students heart-healthy living. b. Participate in blood pressure screenings at the mall. c. Provide pamphlets on heart disease at the grocery store. d. Set up an Ask the nurse booth at the pet store.

ANS: B An important goal of HP2020 is to increase the proportion of adults who have had their blood pressure measured within the preceding 2 years and can state whether their blood pressure was normal or high. Participating in blood pressure screening in a public spot will best help meet that goal. The other options are all appropriate but do not specifically help meet a goal.

A nurse is caring for four clients. Which one should the nurse see first? a. Client who needs a beta blocker, and has a blood pressure of 92/58 mm Hg b. Client who had a first dose of captopril (Capoten) and needs to use the bathroom c. Hypertensive client with a blood pressure of 188/92 mm Hg d. Client who needs pain medication prior to a dressing change of a surgical wound

ANS: B Angiotensin-converting enzyme inhibitors such as captopril can cause hypotension, especially after the first dose. The nurse should see this client first to prevent falling if the client decides to get up without assistance. The two blood pressure readings are abnormal but not critical. The nurse should check on the client with higher blood pressure next to assess for problems related to the reading. The nurse can administer the beta blocker as standards state to hold it if the systolic blood pressure is below 90 mm Hg. The client who needs pain medication prior to the dressing change is not a priority over client safety and assisting the other client to the bathroom.

A nurse cares for a client with arthritis who reports frequent asthma attacks. Which action should the nurse take first? a. Review the clients pulmonary function test results. b. Ask about medications the client is currently taking. c. Assess how frequently the client uses a bronchodilator. d. Consult the provider and request arterial blood gases.

ANS: B Aspirin and other nonsteroidal anti-inflammatory drugs (NSAIDs) can trigger asthma in some people. This results from increased production of leukotriene when aspirin or NSAIDs suppress other inflammatory pathways and is a high priority given the clients history. Reviewing pulmonary function test results will not address the immediate problem of frequent asthma attacks. This is a good intervention for reviewing response to bronchodilators. Questioning the client about the use of bronchodilators will address interventions for the attacks but not their cause. Reviewing arterial blood gas results would not be of use in a client between attacks because many clients are asymptomatic when not having attacks.

A client had a percutaneous transluminal coronary angioplasty for peripheral arterial disease. What assessment finding by the nurse indicates a priority outcome for this client has been met? a. Pain rated as 2/10 after medication b. Distal pulse on affected extremity 2+/4+ c. Remains on bedrest as directed d. Verbalizes understanding of procedure

ANS: B Assessing circulation distal to the puncture site is a critical nursing action. A pulse of 2+/4+ indicates good perfusion. Pain control, remaining on bedrest as directed after the procedure, and understanding are all important, but do not take priority over perfusion.

A nurse administers medications to a client who has asthma. Which medication classification is paired correctly with its physiologic response to the medication? a. Bronchodilator Stabilizes the membranes of mast cells and prevents the release of inflammatory mediators b. Cholinergic antagonist Causes bronchodilation by inhibiting the parasympathetic nervous system c. Corticosteroid Relaxes bronchiolar smooth muscles by binding to and activating pulmonary beta2 receptors d. Cromone Disrupts the production of pathways of inflammatory mediators

ANS: B Cholinergic antagonist drugs cause bronchodilation by inhibiting the parasympathetic nervous system. This allows the sympathetic nervous system to dominate and release norepinephrine that actives beta2 receptors. Bronchodilators relax bronchiolar smooth muscles by binding to and activating pulmonary beta2 receptors. Corticosteroids disrupt the production of pathways of inflammatory mediators. Cromones stabilize the membranes of mast cells and prevent the release of inflammatory mediators.

A nurse assesses a client with mitral valve stenosis. What clinical manifestation should alert the nurse to the possibility that the clients stenosis has progressed? a. Oxygen saturation of 92% b. Dyspnea on exertion c. Muted systolic murmur d. Upper extremity weakness

ANS: B Dyspnea on exertion develops as the mitral valvular orifice narrows and pressure in the lungs increases. The other manifestations do not relate to the progression of mitral valve stenosis.

A home health care nurse is visiting an older client who lives alone after being discharged from the hospital after a coronary artery bypass graft. What finding in the home most causes the nurse to consider additional referrals? a. Dirty carpets in need of vacuuming b. Expired food in the refrigerator c. Old medications in the kitchen d. Several cats present in the home

ANS: B Expired food in the refrigerator demonstrates a safety concern for the client and a possible lack of money to buy food. The nurse can consider a referral to Meals on Wheels or another home-based food program. Dirty carpets may indicate the client has no household help and is waiting for clearance to vacuum. Old medications can be managed by the home health care nurse and the client working collaboratively. Having pets is not a cause for concern.

After assessing a client who is receiving an amiodarone intravenous infusion for unstable ventricular tachycardia, the nurse documents the findings and compares these with the previous assessment findings: Vital Signs Nursing AssessmentTime: 0800 Temperature: 98 FHeart rate: 68 beats/minBlood pressure: 135/60 mm Hg Respiratory rate: 14 breaths/min Oxygen saturation: 96%Oxygen therapy: 2 L nasal cannula Time: 1000Temperature: 98.2 FHeart rate: 50 beats/minBlood pressure: 132/57 mm HgRespiratory rate: 16 breaths/minOxygen saturation: 95%Oxygen therapy: 2 L nasal cannula Time: 0800 Client alert and oriented.Cardiac rhythm: normal sinus rhythm.Skin: warm, dry, and appropriate for race. Respirations equal and unlabored.Client denies shortness of breath and chest pain. Time: 1000Client alert and oriented.Cardiac rhythm: sinus bradycardia.Skin: warm, dry, and appropriate for race. Respirations equal and unlabored.Client denies shortness of breath and chest pain. Client voids 420 mL of clear yellow urine.Based on the assessments, which action should the nurse take? a. Stop the infusion and flush the IV. b. Slow the amiodarone infusion rate. c. Administer IV normal saline. d. Ask the client to cough and deep breathe.

ANS: B IV administration of amiodarone may cause bradycardia and atrioventricular (AV) block. The correct action for the nurse to take at this time is to slow the infusion, because the client is asymptomatic and no evidence reveals AV block that might require pacing. Abruptly ceasing the medication could allow fatal dysrhythmias to occur. The administration of IV fluids and encouragement of coughing and deep breathing exercises are not indicated, and will not increase the clients heart rate.

While assessing a client who is 12 hours postoperative after a thoracotomy for lung cancer, a nurse notices that the lower chest tube is dislodged. Which action should the nurse take first? a. Assess for drainage from the site. b. Cover the insertion site with sterile gauze. c. Contact the provider and obtain a suture kit. d. Reinsert the tube using sterile technique.

ANS: B Immediately covering the insertion site helps prevent air from entering the pleural space and causing a pneumothorax. The area will not reseal quickly enough to prevent air from entering the chest. The nurse should not leave the client to obtain a suture kit. An occlusive dressing may cause a tension pneumothorax. The site should only be assessed after the insertion site is covered. The provider should be called to reinsert the chest tube or prescribe other treatment options.

A student is caring for a client who suffered massive blood loss after trauma. How does the student correlate the blood loss with the clients mean arterial pressure (MAP)? a. It causes vasoconstriction and increased MAP. b. Lower blood volume lowers MAP. c. There is no direct correlation to MAP. d. It raises cardiac output and MAP.

ANS: B Lower blood volume will decrease MAP. The other answers are not accurate.

A client with coronary artery disease (CAD) asks the nurse about taking fish oil supplements. What response by the nurse is best? a. Fish oil is contraindicated with most drugs for CAD. b. The best source is fish, but pills have benefits too. c. There is no evidence to support fish oil use with CAD. d. You can reverse CAD totally with diet and supplements.

ANS: B Omega-3 fatty acids have shown benefit in reducing lipid levels, in reducing the incidence of sudden cardiac death, and for stabilizing atherosclerotic plaque. The best source is fish three times a week or some fish oil supplements. The other options are not accurate.

A client is 4 hours postoperative after a femoropopliteal bypass. The client reports throbbing leg pain on the affected side, rated as 7/10. What action by the nurse takes priority? a. Administer pain medication as ordered. b. Assess distal pulses and skin color. c. Document the findings in the clients chart. d. Notify the surgeon immediately.

ANS: B Once perfusion has been restored or improved to an extremity, clients can often feel a throbbing pain due to the increased blood flow. However, it is important to differentiate this pain from ischemia. The nurse should assess for other signs of perfusion, such as distal pulses and skin color/temperature. Administering pain medication is done once the nurse determines the clients perfusion status is normal. Documentation needs to be thorough. Notifying the surgeon is not necessary.

A nurse teaches a client to use a room humidifier after a laryngectomy. Which statement should the nurse include in this clients teaching? a. Add peppermint oil to the humidifier to relax the airway. b. Make sure you clean the humidifier to prevent infection. c. Keep the humidifier filled with water at all times. d. Use the humidifier when you sleep, even during daytime naps.

ANS: B Priority teaching related to the use of a room humidifier focuses on infection control. Clients should be taught to meticulously clean the humidifier to prevent the spread of mold or other sources of infection. Peppermint oil should not be added to a humidifier. The humidifier should be refilled with water as needed and should be used while awake and asleep.

A nurse is caring for a client after surgery. The clients respiratory rate has increased from 12 to 18 breaths/min and the pulse rate increased from 86 to 98 beats/min since they were last assessed 4 hours ago. What action by the nurse is best? a. Ask if the client needs pain medication. b. Assess the clients tissue perfusion further. c. Document the findings in the clients chart. d. Increase the rate of the clients IV infusion.

ANS: B Signs of the earliest stage of shock are subtle and may manifest in slight increases in heart rate, respiratory rate, or blood pressure. Even though these readings are not out of the normal range, the nurse should conduct a thorough assessment of the client, focusing on indicators of perfusion. The client may need pain medication, but this is not the priority at this time. Documentation should be done thoroughly but is not the priority either. The nurse should not increase the rate of the IV infusion without an order.

A nurse assesses a client with pericarditis. Which assessment finding should the nurse expect to find? a. Heart rate that speeds up and slows down b. Friction rub at the left lower sternal border c. Presence of a regular gallop rhythm d. Coarse crackles in bilateral lung bases

ANS: B The client with pericarditis may present with a pericardial friction rub at the left lower sternal border. This sound is the result of friction from inflamed pericardial layers when they rub together. The other assessments are not related.

A nurse is assessing clients on a medical-surgical unit. Which client should the nurse identify as being at greatest risk for atrial fibrillation? a. A 45-year-old who takes an aspirin daily b. A 50-year-old who is post coronary artery bypass graft surgery c. A 78-year-old who had a carotid endarterectomy d. An 80-year-old with chronic obstructive pulmonary disease

ANS: B Atrial fibrillation occurs commonly in clients with cardiac disease and is a common occurrence after coronary artery bypass graft surgery. The other conditions do not place these clients at higher risk for atrial fibrillation.

The nurse is caring for a client on the medical-surgical unit who suddenly becomes unresponsive and has no pulse. The cardiac monitor shows the rhythm below: After calling for assistance and a defibrillator, which action should the nurse take next? a. Perform a pericardial thump. b. Initiate cardiopulmonary resuscitation (CPR). c. Start an 18-gauge intravenous line. d. Ask the clients family about code status.

ANS: B The clients rhythm is ventricular fibrillation. This is a lethal rhythm that is best treated with immediate defibrillation. While the nurse is waiting for the defibrillator to arrive, the nurse should start CPR. A pericardial thump is not a treatment for ventricular fibrillation. If the client does not already have an IV, other members of the team can insert one after defibrillation. The clients code status should already be known by the nurse prior to this event.

The nurse is evaluating a 3-day diet history with a client who has an elevated lipid panel. What meal selection indicates the client is managing this condition well with diet? a. A 4-ounce steak, French fries, iceberg lettuce b. Baked chicken breast, broccoli, tomatoes c. Fried catfish, cornbread, peas d. Spaghetti with meat sauce, garlic bread

ANS: B The diet recommended for this client would be low in saturated fats and red meat, high in vegetables and whole grains (fiber), low in salt, and low in trans fat. The best choice is the chicken with broccoli and tomatoes. The French fries have too much fat and the iceberg lettuce has little fiber. The catfish is fried. The spaghetti dinner has too much red meat and no vegetables.

After teaching a client how to perform diaphragmatic breathing, the nurse assesses the clients understanding. Which action demonstrates that the client correctly understands the teaching? a. The client lays on his or her side with his or her knees bent. b. The client places his or her hands on his or her abdomen. c. The client lays in a prone position with his or her legs straight. d. The client places his or her hands above his or her head.

ANS: B To perform diaphragmatic breathing correctly, the client should place his or her hands on his or her abdomen to create resistance. This type of breathing cannot be performed effectively while lying on the side or with hands over the head. This type of breathing would not be as effective lying prone

A nurse is caring for a client after surgery who is restless and apprehensive. The unlicensed assistive personnel (UAP) reports the vital signs and the nurse sees they are only slightly different from previous readings. What action does the nurse delegate next to the UAP? a. Assess the client for pain or discomfort. b. Measure urine output from the catheter. c. Reposition the client to the unaffected side. d. Stay with the client and reassure him or her.

ANS: B Urine output changes are a sensitive early indicator of shock. The nurse should delegate emptying the urinary catheter and measuring output to the UAP as a baseline for hourly urine output measurements. The UAP cannot assess for pain. Repositioning may or may not be effective for decreasing restlessness, but does not take priority over physical assessments. Reassurance is a therapeutic nursing action, but the nurse needs to do more in this situation.

A nurse is caring for a client with a deep vein thrombosis (DVT). What nursing assessment indicates a priority outcome has been met? a. Ambulates with assistance b. Oxygen saturation of 98% c. Pain of 2/10 after medication d. Verbalizing risk factors

ANS: B A critical complication of DVT is pulmonary embolism. A normal oxygen saturation indicates that this has not occurred. The other assessments are also positive, but not the priority.

A client has intra-arterial blood pressure monitoring after a myocardial infarction. The nurse notes the clients heart rate has increased from 88 to 110 beats/min, and the blood pressure dropped from 120/82 to 100/60 mm Hg. What action by the nurse is most appropriate? a. Allow the client to rest quietly. b. Assess the client for bleeding. c. Document the findings in the chart. d. Medicate the client for pain.

ANS: B A major complication related to intra-arterial blood pressure monitoring is hemorrhage from the insertion site. Since these vital signs are out of the normal range, are a change, and are consistent with blood loss, the nurse should assess the client for any bleeding associated with the arterial line. The nurse should document the findings after a full assessment. The client may or may not need pain medication and rest; the nurse first needs to rule out any emergent bleeding.

A nurse assesses a client admitted with deep partial-thickness and full-thickness burns on the face, arms, and chest. Which assessment finding should alert the nurse to a potential complication? a. Partial pressure of arterial oxygen (PaO2) of 80 mm Hg b. Urine output of 20 mL/hr c. Productive cough with white pulmonary secretions d. Core temperature of 100.6 F (38 C)

ANS: B A significant loss of fluid occurs with burn injuries, and fluids must be replaced to maintain hemodynamics. If fluid replacement is not adequate, the client may become hypotensive and have decreased perfusion of organs, including the brain and kidneys. A low urine output is an indication of poor kidney perfusion. The other manifestations are not complications of burn injuries.

A client received tissue plasminogen activator (t-PA) after a myocardial infarction and now is on an intravenous infusion of heparin. The clients spouse asks why the client needs this medication. What response by the nurse is best? a. The t-PA didnt dissolve the entire coronary clot. b. The heparin keeps that artery from getting blocked again. c. Heparin keeps the blood as thin as possible for a longer time. d. The heparin prevents a stroke from occurring as the t-PA wears off.

ANS: B After the original intracoronary clot has dissolved, large amounts of thrombin are released into the bloodstream, increasing the chance of the vessel reoccluding. The other statements are not accurate. Heparin is not a blood thinner, although laypeople may refer to it as such.

A nurse assesses a client who has mitral valve regurgitation. For which cardiac dysrhythmia should the nurse assess? a. Preventricular contractions b. Atrial fibrillation c. Symptomatic bradycardia d. Sinus tachycardia

ANS: B Atrial fibrillation is a clinical manifestation of mitral valve regurgitation and stenosis. Preventricular contractions and bradycardia are not associated with valvular problems. These are usually identified in clients with electrolyte imbalances, myocardial infarction, and sinus node problems. Sinus tachycardia is a manifestation of aortic regurgitation due to a decrease in cardiac output.

A client has been brought to the emergency department with a life-threatening chest injury. What action by the nurse takes priority? A. Apply oxygen at 100%. B. Assess the respiratory rate. C. Ensure a patent airway. D. Start two large-bore IV lines.

ANS: C The priority for any chest trauma client is airway, breathing, circulation. The nurse first ensures the client has a patent airway. Assessing respiratory rate and applying oxygen are next, followed by inserting IVs.

A nurse is assessing clients on a medical-surgical unit. Which client would the nurse identify as being at greatest risk for atrial fibrillation? a. A 45-year-old who takes an aspirin daily. b. A 50-year-old who is post coronary artery bypass graft surgery. c. A 78-year-old who had a carotid endarterectomy. d. An 80-year-old with chronic obstructive pulmonary disease.

ANS: B Atrial fibrillation occurs commonly in clients with cardiac disease. Other risk factors include hypertension (HTN), previous ischemic stroke, transient ischemic attack (TIA) or other thromboembolic event, diabetes mellitus, heart failure, obesity, hyperthyroidism, chronic kidney disease, excessive alcohol use, and mitral valve disease. The other conditions do not place these clients at higher risk for atrial fibrillation.

A nurse evaluates prescriptions for a client with chronic atrial fibrillation. Which medication would the nurse expect to find on this client's medication administration record to prevent a common complication of this condition? a. Sotalol b. Warfarin c. Atropine d. Lidocaine

ANS: B Atrial fibrillation puts clients at risk for developing emboli. Clients at risk for emboli are treated with anticoagulants, such as heparin, enoxaparin, or warfarin. Sotalol, atropine, and lidocaine are not appropriate for preventing this complication.

A nurse evaluates prescriptions for a client with chronic atrial fibrillation. Which medication should the nurse expect to find on this clients medication administration record to prevent a common complication of this condition? a. Sotalol (Betapace) b. Warfarin (Coumadin) c. Atropine (Sal-Tropine) d. Lidocaine (Xylocaine)

ANS: B Atrial fibrillation puts clients at risk for developing emboli. Clients at risk for emboli are treated with anticoagulants, such as heparin, enoxaparin, or warfarin. Sotalol, atropine, and lidocaine are not appropriate for this complication.

The nurse is caring for a client with an acute burn injury. Which action should the nurse take to prevent infection by autocontamination? a. Use a disposable blood pressure cuff to avoid sharing with other clients. b. Change gloves between wound care on different parts of the clients body. c. Use the closed method of burn wound management for all wound care. d. Advocate for proper and consistent handwashing by all members of the staff.

ANS: B Autocontamination is the transfer of microorganisms from one area to another area of the same clients body, causing infection of a previously uninfected area. Although all techniques listed can help reduce the risk for infection, only changing gloves between performing wound care on different parts of the clients body can prevent autocontamination.

A nurse cares for a client who has a heart rate averaging 56 beats/min with no adverse symptoms. Which activity modification would the nurse suggest to avoid further slowing of the heart rate? a. "Make certain that your bath water is warm." b. "Avoid straining while having a bowel movement." c. "Limit your intake of caffeinated drinks to one a day." d. "Avoid strenuous exercise such as running."

ANS: B Bearing down strenuously during a bowel movement is one type of Valsalva maneuver, which stimulates the vagus nerve and results in slowing of the heart rate. Such a response is not desirable in a person who has bradycardia. The other instructions are not appropriate for this condition.

A nurse cares for a client who has a heart rate averaging 56 beats/min with no adverse symptoms. Which activity modification should the nurse suggest to avoid further slowing of the heart rate? a. Make certain that your bath water is warm. b. Avoid straining while having a bowel movement. c. Limit your intake of caffeinated drinks to one a day. d. Avoid strenuous exercise such as running.

ANS: B Bearing down strenuously during a bowel movement is one type of Valsalva maneuver, which stimulates the vagus nerve and results in slowing of the heart rate. Such a response is not desirable in a person who has bradycardia. The other instructions are not appropriate for this condition.

A nurse teaches a client with diabetes mellitus and a body mass index of 42 who is at high risk for coronary artery disease. Which statement related to nutrition should the nurse include in this clients teaching? a. The best way to lose weight is a high-protein, low-carbohydrate diet. b. You should balance weight loss with consuming necessary nutrients. c. A nutritionist will provide you with information about your new diet. d. If you exercise more frequently, you wont need to change your diet.

ANS: B Clients at risk for cardiovascular diseases should follow the American Heart Association guidelines to combat obesity and improve cardiac health. The nurse should encourage the client to eat vegetables, fruits, unrefined whole-grain products, and fat-free dairy products while losing weight. High-protein food items are often high in fat and calories. Although the nutritionist can assist with client education, the nurse should include nutrition education and assist the client to make healthy decisions. Exercising and eating nutrient-rich foods are both important components in reducing cardiovascular risk.

A client is receiving an infusion of alteplase (Activase) for an intra-arterial clot. The client begins to mumble and is disoriented. What action by the nurse takes priority? a. Assess the clients neurologic status. b. Notify the Rapid Response Team. c. Prepare to administer vitamin K. d. Turn down the infusion rate.

ANS: B Clients on fibrinolytic therapy are at high risk of bleeding. The sudden onset of neurologic signs may indicate the client is having a hemorrhagic stroke. The nurse does need to complete a thorough neurological examination, but should first call the Rapid Response Team based on the clients manifestations. The nurse notifies the Rapid Response Team first. Vitamin K is not the antidote for this drug. Turning down the infusion rate will not be helpful if the client is still receiving any of the drug.

After teaching a client who is being discharged home after mitral valve replacement surgery, the nurse assesses the clients understanding. Which client statement indicates a need for additional teaching? a. Ill be able to carry heavy loads after 6 months of rest. b. I will have my teeth cleaned by my dentist in 2 weeks. c. I must avoid eating foods high in vitamin K, like spinach. d. I must use an electric razor instead of a straight razor to shave.

ANS: B Clients who have defective or repaired valves are at high risk for endocarditis. The client who has had valve surgery should avoid dental procedures for 6 months because of the risk for endocarditis. When undergoing a mitral valve replacement surgery, the client needs to be placed on anticoagulant therapy to prevent vegetation forming on the new valve. Clients on anticoagulant therapy should be instructed on bleeding precautions, including using an electric razor. If the client is prescribed warfarin, the client should avoid foods high in vitamin K. Clients recovering from open heart valve replacements should not carry anything heavy for 6 months while the chest incision and muscle heal.

A nurse assesses a client who has a history of heart failure. Which question should the nurse ask to assess the extent of the clients heart failure? a. Do you have trouble breathing or chest pain? b. Are you able to walk upstairs without fatigue? c. Do you awake with breathlessness during the night? d. Do you have new-onset heaviness in your legs?

ANS: B Clients with a history of heart failure generally have negative findings, such as shortness of breath. The nurse needs to determine whether the clients activity is the same or worse, or whether the client identifies a decrease in activity level. Trouble breathing, chest pain, breathlessness at night, and peripheral edema are symptoms of heart failure, but do not provide data that can determine the extent of the clients heart failure.

A nurse assesses a client with atrial fibrillation. Which manifestation would alert the nurse to the possibility of a serious complication from this condition? a. Sinus tachycardia b. Speech alterations c. Fatigue d. Dyspnea with activity

ANS: B Clients with atrial fibrillation are at risk for embolic stroke. Evidence of embolic events includes changes in mentation, speech, sensory function, and motor function. Clients with atrial fibrillation often have a rapid ventricular response as a result. Fatigue is a nonspecific complaint. Clients with atrial fibrillation often have dyspnea as a result of the decreased cardiac output caused by the rhythm disturbance.

A nurse assesses a client with atrial fibrillation. Which manifestation should alert the nurse to the possibility of a serious complication from this condition? a. Sinus tachycardia b. Speech alterations c. Fatigued. d. Dyspnea with activity

ANS: B Clients with atrial fibrillation are at risk for embolic stroke. Evidence of embolic events includes changes in mentation, speech, sensory function, and motor function. Clients with atrial fibrillation often have a rapid ventricular response as a result. Fatigue is a nonspecific complaint. Clients with atrial fibrillation often have dyspnea as a result of the decreased cardiac output caused by the rhythm disturbance.

A student nurse is preparing to administer enoxaparin (Lovenox) to a client. What action by the student requires immediate intervention by the supervising nurse? A. Assessing the clients platelet count B. Choosing an 18-gauge, 2-inch needle C. Not aspirating prior to injection D. Swabbing the injection site with alcohol

ANS: B Enoxaparin is given subcutaneously, so the 18-gauge, 2-inch needle is too big. The other actions are appropriate.

A client is on intravenous heparin to treat a pulmonary embolism. The clients most recent partial thromboplastin time (PTT) was 25 seconds. What order should the nurse anticipate? A. Decrease the heparin rate. B. Increase the heparin rate. C. No change to the heparin rate. D. Stop heparin; start warfarin (Coumadin).

ANS: B For clients on heparin, a PTT of 1.5 to 2.5 times the normal value is needed to demonstrate the heparin is working. A normal PTT is 25 to 35 seconds, so this clients PTT value is too low. The heparin rate needs to be increased. Warfarin is not indicated in this situation.

A nurse assists with the cardioversion of a client experiencing acute atrial fibrillation. What action would the nurse take prior to the cardioversion? a. Administer intravenous adenosine. b. Turn off oxygen therapy. c. Ensure that a tongue blade is available. d. Position the client on the left side.

ANS: B For safety during cardioversion, the nurse would turn off any oxygen therapy to prevent fire. The other interventions are not appropriate for a cardioversion. The client would be placed in a supine position.

A nurse is preparing to admit a client on mechanical ventilation from the emergency department. What action by the nurse takes priority? A. Assessing that the ventilator settings are correct B. Ensuring there is a bag-valve-mask in the room C. Obtaining personal protective equipment D. Planning to suction the client upon arrival to the room

ANS: B Having a bag-valve-mask device is critical in case the client needs manual breathing. The respiratory therapist is usually primarily responsible for setting up the ventilator, although the nurse should know and check the settings. Personal protective equipment is important, but ensuring client safety takes priority. The client may or may not need suctioning on arrival.

A nurse is in charge of the coronary intensive care unit. Which client should the nurse see first? a. Client on a nitroglycerin infusion at 5 mcg/min, not titrated in the last 4 hours b. Client who is 1 day post coronary artery bypass graft, blood pressure 180/100 mm Hg c. Client who is 1 day post percutaneous coronary intervention, going home this morning d. Client who is 2 days post coronary artery bypass graft, became dizzy this a.m. while walking

ANS: B Hypertension after coronary artery bypass graft surgery can be dangerous because it puts too much pressure on the suture lines and can cause bleeding. The charge nurse should see this client first. The client who became dizzy earlier should be seen next. The client on the nitroglycerin drip is stable. The client going home can wait until the other clients are cared for.

A client has hemodynamic monitoring after a myocardial infarction. What safety precaution does the nurse implement for this client? a. Document pulmonary artery wedge pressure (PAWP) readings and assess their trends. b. Ensure the balloon does not remain wedged. c. Keep the client on strict NPO status. d. Maintain the client in a semi-Fowlers position.

ANS: B If the balloon remains inflated, it can cause pulmonary infarction or rupture. The nurse should ensure the balloon remains deflated between PAWP readings. Documenting PAWP readings and assessing trends is an important nursing action related to hemodynamic monitoring, but is not specifically related to safety. The client does not have to be NPO while undergoing hemodynamic monitoring. Positioning may or may not affect readings.

The nurse is caring for a client with a chest tube after a coronary artery bypass graft. The drainage slows significantly. What action by the nurse is most important? a. Increase the setting on the suction. b. Notify the provider immediately. c. Re-position the chest tube. d. Take the tubing apart to assess for clots.

ANS: B If the drainage in the chest tube decreases significantly and dramatically, the tube may be blocked by a clot. This could lead to cardiac tamponade. The nurse should notify the provider immediately. The nurse should not independently increase the suction, re-position the chest tube, or take the tubing apart.

A nurse cares for a client with an intravenous temporary pacemaker for bradycardia. The nurse observes the presence of a pacing spike but no QRS complex on the client's electrocardiogram. What action would the nurse take next? a. Administer intravenous diltiazem. b. Assess vital signs and level of consciousness. c. Administer sublingual nitroglycerin. d. Assess capillary refill and temperature.

ANS: B In temporary pacing, the wires are threaded onto the epicardial surface of the heart and exit through the chest wall. The pacemaker spike would be followed immediately by a QRS complex. Pacing spikes seen without subsequent QRS complexes imply loss of capture. If there is no capture, then there is no ventricular depolarization and contraction. The nurse would assess for cardiac output via vital signs and level of consciousness. The other interventions would not determine if the client is tolerating the loss of capture.

A nurse cares for a client with an intravenous temporary pacemaker for bradycardia. The nurse observes the presence of a pacing spike but no QRS complex on the clients electrocardiogram. Which action should the nurse take next? a. Administer intravenous diltiazem (Cardizem). b. Assess vital signs and level of consciousness. c. Administer sublingual nitroglycerin. d. Assess capillary refill and temperature.

ANS: B In temporary pacing, the wires are threaded onto the epicardial surface of the heart and exit through the chest wall. The pacemaker spike should be followed immediately by a QRS complex. Pacing spikes seen without subsequent QRS complexes imply loss of capture. If there is no capture, then there is no ventricular depolarization and contraction. The nurse should assess for cardiac output via vital signs and level of consciousness. The other interventions would not determine if the client is tolerating the loss of capture.

A nurse cares for a client who had a partial laryngectomy 10 days ago. The client states that all food tastes bland. How should the nurse respond? a. I will consult the speech therapist to ensure you are swallowing properly. b. This is normal after surgery. What types of food do you like to eat? c. I will ask the dietitian to change the consistency of the food in your diet. d. Replacement of protein, calories, and water is very important after surgery.

ANS: B Many clients experience changes in taste after surgery. The nurse should identify foods that the client wants to eat to ensure the client maintains necessary nutrition. Although the nurse should collaborate with the speech therapist and dietitian to ensure appropriate replacement of protein, calories, and water, the other responses do not address the clients concerns.

A client has been diagnosed with hypertension but does not take the antihypertensive medications because of a lack of symptoms. What response by the nurse is best? a. Do you have trouble affording your medications?b. Most people with hypertension do not have symptoms. c. You are lucky; most people get severe morning headaches. d. You need to take your medicine or you will get kidney failure.

ANS: B Most people with hypertension are asymptomatic, although a small percentage do have symptoms such as headache. The nurse should explain this to the client. Asking about paying for medications is not related because the client has already admitted nonadherence. Threatening the client with possible complications will not increase compliance.

A client is hospitalized with a second episode of pulmonary embolism (PE). Recent genetic testing reveals the client has an alteration in the gene CYP2C19. What action by the nurse is best? A.Instruct the client to eliminate all vitamin K from the diet. B. Prepare preoperative teaching for an inferior vena cava (IVC) filter. C. Refer the client to a chronic illness support group. D. Teach the client to use a soft-bristled toothbrush.

ANS: B Often clients are discharged from the hospital on warfarin (Coumadin) after a PE. However, clients with a variation in the CYP2C19 gene do not metabolize warfarin well and have higher blood levels and more side effects. This client is a poor candidate for warfarin therapy, and the prescriber will most likely order an IVC ilter device to be implanted. The nurse should prepare to do preoperative teaching on this procedure. It would be impossible to eliminate all vitamin K from the diet. A chronic illness support group may be needed, but this is not the best intervention as it is not as specific to the client as the IVC filter. A soft-bristled toothbrush is a safety measure for clients on anticoagulation therapy.

A nurse works at a community center for older adults. What self-management measure can the nurse teach the clients to prevent shock? a. Do not get dehydrated in warm weather. b. Drink fluids on a regular schedule. c. Seek attention for any lacerations. d. Take medications as prescribed.

ANS: B Preventing dehydration in older adults is important because the age-related decrease in the thirst mechanism makes them prone to dehydration. Having older adults drink fluids on a regular schedule will help keep them hydrated without the influence of thirst (or lack of thirst). Telling clients not to get dehydrated is important, but not the best answer because it doesnt give them the tools to prevent it from occurring. Older adults should seek attention for lacerations, but this is not as important an issue as staying hydrated. Taking medications as prescribed may or may not be related to hydration.

A nurse assesses a client admitted to the cardiac unit. Which statement by the client alerts the nurse to the possibility of right-sided heart failure? a. I sleep with four pillows at night. b. My shoes fit really tight lately. c. I wake up coughing every night. d. I have trouble catching my breath.

ANS: B Signs of systemic congestion occur with right-sided heart failure. Fluid is retained, pressure builds in the venous system, and peripheral edema develops. Left-sided heart failure symptoms include respiratory symptoms. Orthopnea, coughing, and difficulty breathing all could be results of left-sided heart failure.

A nurse cares for a client recovering from prosthetic valve replacement surgery. The client asks, Why will I need to take anticoagulants for the rest of my life? How should the nurse respond? a. The prosthetic valve places you at greater risk for a heart attack. b. Blood clots form more easily in artificial replacement valves. c. The vein taken from your leg reduces circulation in the leg. d. The surgery left a lot of small clots in your heart and lungs.

ANS: B Synthetic valve prostheses and scar tissue provide surfaces on which platelets can aggregate easily and initiate the formation of blood clots. The other responses are inaccurate.

A client has presented to the emergency department with an acute myocardial infarction (MI). What action by the nurse is best to meet The Joint Commissions Core Measures outcomes? a. Obtain an electrocardiogram (ECG) now and in the morning. b. Give the client an aspirin. c. Notify the Rapid Response Team. d. Prepare to administer thrombolytics.

ANS: B The Joint Commissions Core Measures set for acute MI require that aspirin is administered when a client with MI presents to the emergency department or when an MI occurs in the hospital. A rapid ECG is vital, but getting another one in the morning is not part of the Core Measures set. The Rapid Response Team is not needed if an emergency department provider is available. Thrombolytics may or may not be needed.

A nurse cares for a client who has an 80% blockage of the right coronary artery (RCA) and is scheduled for bypass surgery. Which intervention should the nurse be prepared to implement while this client waits for surgery? a. Administration of IV furosemide (Lasix) b. Initiation of an external pacemaker c. Assistance with endotracheal intubation d. Placement of central venous access.

ANS: B The RCA supplies the right atrium, the right ventricle, the inferior portion of the left ventricle, and the atrioventricular (AV) node. It also supplies the sinoatrial node in 50% of people. If the client totally occludes the RCA, the AV node would not function and the client would go into heart block, so emergency pacing should be available for the client. Furosemide, intubation, and central venous access will not address the primary complication of RCA occlusion, which is AV node malfunction

After teaching a client who has an implantable cardioverter-defibrillator (ICD), a nurse assesses the client's understanding. Which statement by the client indicates correct understanding of the teaching? a. "I would wear a snug-fitting shirt over the ICD." b. "I will avoid sources of strong electromagnetic fields." c. "I would participate in a strenuous exercise program." d. "Now I can discontinue my antidysrhythmic medication."

ANS: B The client being discharged with an ICD is instructed to avoid strong sources of electromagnetic fields, such as devices emitting microwaves (not microwave ovens); transformers; radio, television, and radar transmitters; large electrical generators; metal detectors, including handheld security devices at airports; antitheft devices; arc welding equipment; and sources of 60-cycle (Hz) interference. Also avoid leaning directly over the alternator of a running motor of a car or boat. Clients would avoid tight clothing, which could cause irritation over the ICD generator. The client would be encouraged to exercise but would not engage in strenuous activities that cause the heart rate to meet or exceed the ICD cutoff point because the ICD can discharge inappropriately. The client would continue all prescribed medications.

After teaching a client who has an implantable cardioverter-defibrillator (ICD), a nurse assesses the clients understanding. Which statement by the client indicates a correct understanding of the teaching? a. I should wear a snug-fitting shirt over the ICD. b. I will avoid sources of strong electromagnetic fields. c. I should participate in a strenuous exercise program. d. Now I can discontinue my antidysrhythmic medication.

ANS: B The client being discharged with an ICD is instructed to avoid strong sources of electromagnetic fields. Clients should avoid tight clothing, which could cause irritation over the ICD generator. The client should be encouraged to exercise but should not engage in strenuous activities that cause the heart rate to meet or exceed the ICD cutoff point because the ICD can discharge inappropriately. The client should continue all prescribed medications.

A pulmonary nurse cares for clients who have chronic obstructive pulmonary disease (COPD). Which client should the nurse assess first? a. A 46-year-old with a 30pack-year history of smoking b. A 52-year-old in a tripod position using accessory muscles to breathe c. A 68-year-old who has dependent edema and clubbed fingers d. A 74-year-old with a chronic cough and thick, tenacious secretions

ANS: B The client who is in a tripod position and using accessory muscles is working to breathe. This client must be assessed first to establish how well the client is breathing and provide interventions to minimize respiratory failure. The other clients are not in acute distress.

A nurse teaches a client who has open vocal cord paralysis. Which technique should the nurse teach the client to prevent aspiration? a. Tilt the head back as far as possible when swallowing. b. Tuck the chin down when swallowing. c. Breathe slowly and deeply while swallowing. d. Keep the head very still and straight while swallowing.

ANS: B The client with open vocal cord paralysis may aspirate. The nurse should teach the client to tuck in his or her chin during swallowing to prevent aspiration. Tilting the head back would increase the chance of aspiration. Breathing slowly would not decrease the risk of aspiration, but holding the breath would. Keeping the head still and straight would not decrease the risk for aspiration.

A nurse cares for a client who is prescribed magnetic resonance imaging (MRI) of the heart. The clients health history includes a previous myocardial infarction and pacemaker implantation. Which action should the nurse take? a. Schedule an electrocardiogram just before the MRI. b. Notify the health care provider before scheduling the MRI. c. Call the physician and request a laboratory draw for cardiac enzymes. d. Instruct the client to increase fluid intake the day before the MRI.

ANS: B The magnetic fields of the MRI can deactivate the pacemaker. The nurse should call the health care provider and report that the client has a pacemaker so the provider can order other diagnostic tests. The client does not need an electrocardiogram, cardiac enzymes, or increased fluids.

A nurse receives new prescriptions for a client with severe burn injuries who is receiving fluid resuscitation per the Parkland formula. The clients urine output continues to range from 0.2 to 0.25 mL/kg/hr. Which prescription should the nurse question? a. Increase intravenous fluids by 100 mL/hr. b. Administer furosemide (Lasix) 40 mg IV push. c. Continue to monitor urine output hourly. d. Draw blood for serum electrolytes STAT.

ANS: B The plan of care for a client with a burn includes fluid and electrolyte resuscitation. Furosemide would be inappropriate to administer. Postburn fluid needs are calculated initially by using a standardized formula such as the Parkland formula. However, needs vary among clients, and the final fluid volume needed is adjusted to maintain hourly urine output at 0.5 mL/kg/hr. Based on this clients inadequate urine output, fluids need to be increased, urine output needs to be monitored hourly, and electrolytes should be evaluated to ensure appropriate fluids are being infused.

A nurse is caring for four clients. Which client should the nurse assess first? a. Client with an acute myocardial infarction, pulse 102 beats/min b. Client who is 1 hour post angioplasty, has tongue swelling and anxiety c. Client who is post coronary artery bypass, chest tube drained 100 mL/hr d. Client who is post coronary artery bypass, potassium 4.2 mEq/L

ANS: B The post-angioplasty client with tongue swelling and anxiety is exhibiting manifestations of an allergic reaction that could progress to anaphylaxis. The nurse should assess this client first. The client with a heart rate of 102 beats/min may have increased oxygen demands but is just over the normal limit for heart rate. The two post coronary artery bypass clients are stable.

A nurse auscultated heart tones on an older adult client. Which action should the nurse take based on heart tones heard?(Click the media button to hear the audio clip.) a. Administer a diuretic. b. Document the finding. c. Decrease the IV flow rate. d. Evaluate the clients medications.

ANS: B The sound heard is an atrial gallop S4. An atrial gallop may be heard in older clients because of a stiffened ventricle. The nurse should document the finding, but no other intervention is needed at this time.

A nurse teaches a client recovering from a heart transplant who is prescribed cyclosporine (Sandimmune). Which statement should the nurse include in this clients discharge teaching? a. Use a soft-bristled toothbrush and avoid flossing. b. Avoid large crowds and people who are sick. c. Change positions slowly to avoid hypotension. d. Check your heart rate before taking the medication.

ANS: B These agents cause immune suppression, leaving the client more vulnerable to infection. The medication does not place the client at risk for bleeding, orthostatic hypotension, or a change in heart rate.

A nurse cares for a client with burn injuries from a house fire. The client is not consistently oriented and reports a headache. Which action should the nurse take? a. Increase the clients oxygen and obtain blood gases. b. Draw blood for a carboxyhemoglobin level. c. Increase the clients intravenous fluid rate. d. Perform a thorough Mini-Mental State Examination.

ANS: B These manifestations are consistent with moderated carbon monoxide poisoning. This client is at risk for carbon monoxide poisoning because he or she was in a fire in an enclosed space. The other options will not provide information related to carbon monoxide poisoning.

A nurse answers a call light and finds a client anxious, short of breath, reporting chest pain, and having a blood pressure of 88/52 mm Hg on the cardiac monitor. What action by the nurse takes priority? A. Assess the clients lung sounds. B Notify the Rapid Response Team. C. Provide reassurance to the client. D. Take a full set of vital signs.

ANS: B This client has manifestations of a pulmonary embolism, and the most critical action is to notify the Rapid Response Team for speedy diagnosis and treatment. The other actions are appropriate also but are not the priority.

A client is in the hospital after suffering a myocardial infarction and has bathroom privileges. The nurse assists the client to the bathroom and notes the clients O2 saturation to be 95%, pulse 88 beats/min, and respiratory rate 16 breaths/min after returning to bed. What action by the nurse is best? a. Administer oxygen at 2 L/min. b. Allow continued bathroom privileges. c. Obtain a bedside commode. d. Suggest the client use a bedpan.

ANS: B This clients physiologic parameters did not exceed normal during and after activity, so it is safe for the client to continue using the bathroom. There is no indication that the client needs oxygen, a commode, or a bedpan.

A nurse is caring for four clients on intravenous heparin therapy. Which laboratory value possibly indicates that a serious side effect has occurred? A.Hemoglobin: 14.2 g/dL B. Platelet count: 82,000/L C. Red blood cell count: 4.8/mm3 D. White blood cell count: 8.7/mm3

ANS: B This platelet count is low and could indicate heparin-induced thrombocytopenia. The other values are normal for either gender.

After teaching a client who is prescribed voice rest therapy for vocal cord polyps, a nurse assesses the clients understanding. Which statement indicates the client needs further teaching? a. I will stay away from smokers to minimize inhalation of secondhand smoke. b. When I speak, I will whisper rather than use a normal tone of voice. c. For the next several weeks, I will not lift more than 10 pounds. d. I will drink at least three quarts of water each day to stay hydrated.

ANS: B Treatment for vocal cord polyps includes no speaking, no lifting, and no smoking. The client has to be educated not to even whisper when resting the voice. It is also appropriate for the client to stay out of rooms where people are smoking, to stay hydrated, and to use stool softeners.

A nurse is teaching a client about warfarin (Coumadin). What assessment finding by the nurse indicates a possible barrier to self-management? A. Poor visual acuity B. Strict vegetarian C. Refusal to stop smoking D. Wants weight loss surgery

ANS: B Warfarin works by inhibiting the synthesis of vitamin Kdependent clotting factors. Foods high in vitamin K thus interfere with its action and need to be eaten in moderate, consistent amounts. A vegetarian may have trouble maintaining this diet. The nurse should explore this possibility with the client. The other options are not related.

A client is being discharged soon on warfarin (Coumadin). What menu selection for dinner indicates the client needs more education regarding this medication? A. Hamburger and French fries B. Large chefs salad and muffin C. No selection; spouse brings pizza D. Tuna salad sandwich and chips

ANS: B Warfarin works by inhibiting the synthesis of vitamin Kdependent clotting factors. Foods high in vitamin K thus interfere with its action and need to be eaten in moderate, consistent amounts. The chefs salad most likely has too many leafy green vegetables, which contain high amounts of vitamin K. The other selections, while not particularly healthy, will not interfere with the medications mechanism of action.

A client has a deep vein thrombosis (DVT). What comfort measure does the nurse delegate to the unlicensed assistive personnel (UAP)? a. Ambulate the client. b. Apply a warm moist pack. c. Massage the clients leg. d. Provide an ice pack.

ANS: B Warm moist packs will help with the pain of a DVT. Ambulation is not a comfort measure. Massaging the clients legs is contraindicated to prevent complications such as pulmonary embolism. Ice packs are not recommended for DVT.

A nurse obtains the health history of a client who is newly admitted to the medical unit. Which statement by the client should alert the nurse to the presence of edema? a. I wake up to go to the bathroom at night. b. My shoes fit tighter by the end of the day. c. I seem to be feeling more anxious lately. d. I drink at least eight glasses of water a day.

ANS: B Weight gain can result from fluid accumulation in the interstitial spaces. This is known as edema. The nurse should note whether the client feels that his or her shoes or rings are tight, and should observe, when present, an indentation around the leg where the socks end. The other answers do not describe edema.

A nurse is caring for a client on the medical stepdown unit. The following data are related to this client: Subjective Information: Shortness of breath for 20 minutes Feels frightened Can't catch my breath Laboratory Analysis Physical Assessment: What action by the nurse is most appropriate? A. Call respiratory therapy for a breathing treatment. B. Facilitate a STAT pulmonary angiography. C. Prepare for immediate endotracheal intubation. D. Prepare to administer intravenous anticoagulants.

ANS: B pH: 7.12 PaCO2: 28 mm Hg PaO2: 58 mm Hg SaO2: 88% Pulse: 120 beats/min Respiratory rate: 34 breaths/min Blood pressure 158/92 mm Hg Lungs have crackles. This client has manifestations of pulmonary embolism (PE); however, many conditions can cause the clients presentation. The gold standard for diagnosing a PE is pulmonary angiography. The nurse should facilitate this test as soon as possible. The client does not have wheezing, so a respiratory treatment is not needed. The client is not unstable enough to need intubation and mechanical ventilation. IV anticoagulants are not given without a diagnosis of PE.

The provider requests the nurse start an infusion of an inotropic agent on a client. How does the nurse explain the action of these drugs to the client and spouse? a. It constricts vessels, improving blood flow. b. It dilates vessels, which lessens the work of the heart. c. It increases the force of the hearts contractions. d. It slows the heart rate down for better filling.

ANS: C A positive inotrope is a medication that increases the strength of the hearts contractions. The other options are not correct.

A client in the cardiac stepdown unit reports severe, crushing chest pain accompanied by nausea and vomiting. What action by the nurse takes priority? a. Administer an aspirin. b. Call for an electrocardiogram (ECG). c. Maintain airway patency. d. Notify the provider.

ANS: C Airway always is the priority. The other actions are important in this situation as well, but the nurse should stay with the client and ensure the airway remains patent (especially if vomiting occurs) while another person calls the provider (or Rapid Response Team) and facilitates getting an ECG done. Aspirin will probably be administered, depending on the providers prescription and the clients current medications.

A nurse cares for a client who is infected with Burkholderia cepacia. Which action should the nurse take first when admitting this client to a pulmonary care unit? a. Instruct the client to wash his or her hands after contact with other people. b. Implement Droplet Precautions and don a surgical mask. c. Keep the client isolated from other clients with cystic fibrosis. d. Obtain blood, sputum, and urine culture specimens.

ANS: C Burkholderia cepacia infection is spread through casual contact between cystic fibrosis clients, thus the need for these clients to be separated from one another. Strict isolation measures will not be necessary. Although the client should wash his or her hands frequently, the most important measure that can be implemented on the unit is isolation of the client from other clients with cystic fibrosis. There is no need to implement Droplet Precautions or don a surgical mask when caring for this client. Obtaining blood, sputum, and urine culture specimens will not provide information necessary to care for a client with Burkholderia cepacia infection.

A client in shock has been started on dopamine. What assessment finding requires the nurse to communicate with the provider immediately? a. Blood pressure of 98/68 mm Hg b. Pedal pulses 1+/4+ bilaterally c. Report of chest heaviness d. Urine output of 32 mL/hr

ANS: C Chest heaviness or pain indicates myocardial ischemia, a possible adverse effect of dopamine. While taking dopamine, the oxygen requirements of the heart are increased due to increased myocardial workload, and may cause ischemia. Without knowing the clients previous blood pressure or pedal pulses, there is not enough information to determine if these are an improvement or not. A urine output of 32 mL/hr is acceptable.

The nurse is caring for a client with lung cancer who states, I dont want any pain medication because I am afraid Ill become addicted. How should the nurse respond? a. I will ask the provider to change your medication to a drug that is less potent. b. Would you like me to use music therapy to distract you from your pain? c. It is unlikely you will become addicted when taking medicine for pain. d. Would you like me to give you acetaminophen (Tylenol) instead?

ANS: C Clients should be encouraged to take their pain medications; addiction usually is not an issue with a client in pain. The nurse would not request that the pain medication be changed unless it was not effective. Other methods to decrease pain can be used, in addition to pain medication.

A nurse cares for a client with atrial fibrillation who reports fatigue when completing activities of daily living. What interventions should the nurse implement to address this clients concerns? a. Administer oxygen therapy at 2 liters per nasal cannula. b. Provide the client with a sleeping pill to stimulate rest. c. Schedule periods of exercise and rest during the day. d. Ask unlicensed assistive personnel to help bathe the client.

ANS: C Clients who have atrial fibrillation are at risk for decreased cardiac output and fatigue when completing activities of daily living. The nurse should schedule periods of exercise and rest during the day to decrease fatigue. The other interventions will not assist the client with self-care activities.

The nurse asks a client who has experienced ventricular dysrhythmias about substance abuse. The client asks, Why do you want to know if I use cocaine? How should the nurse respond? a. Substance abuse puts clients at risk for many health issues. b. The hospital requires that I ask you about cocaine use. c. Clients who use cocaine are at risk for fatal dysrhythmias. d. We can provide services for cessation of substance abuse.

ANS: C Clients who use cocaine or illicit inhalants are particularly at risk for potentially fatal dysrhythmias. The other responses do not adequately address the clients question.

The nurse is teaching a client with chronic obstructive pulmonary disease who has been prescribed continuous oxygen therapy at home. Which statement indicates the client correctly understands the teaching? a. I plan to wear my oxygen when I exercise and feel short of breath. b. I will use my portable oxygen when grilling burgers in the backyard. c. I plan to use cotton balls to cushion the oxygen tubing on my ears. d. I will only smoke while I am wearing my oxygen via nasal cannula.

ANS: C Cotton balls can decrease pressure ulcers from the oxygen tubing. Continuous oxygen orders mean the client should wear the oxygen at all times. Oxygen fuels a fire. Wearing oxygen while grilling and smoking increases the risk for fire.

A student nurse asks what essential hypertension is. What response by the registered nurse is best? a. It means it is caused by another disease. b. It means it is essential that it be treated. c. It is hypertension with no specific cause. d. It refers to severe and life-threatening hypertension.

ANS: C Essential hypertension is the most common type of hypertension and has no specific cause such as an underlying disease process. Hypertension that is due to another disease process is called secondary hypertension. A severe, life-threatening form of hypertension is malignant hypertension.

After teaching a client who is prescribed a long-acting beta2 agonist medication, a nurse assesses the clients understanding. Which statement indicates the client comprehends the teaching? a. I will carry this medication with me at all times in case I need it. b. I will take this medication when I start to experience an asthma attack. c. I will take this medication every morning to help prevent an acute attack. d. I will be weaned off this medication when I no longer need it.

ANS: C Long-acting beta2 agonist medications will help prevent an acute asthma attack because they are long acting. The client will take this medication every day for best effect. The client does not have to always keep this medication with him or her because it is not used as a rescue medication. This is not the medication the client will use during an acute asthma attack because it does not have an immediate onset of action. The client will not be weaned off this medication because this is likely to be one of his or her daily medications.

A student nurse is caring for a client who will be receiving sodium nitroprusside (Nipride) via IV infusion. What action by the student causes the registered nurse to intervene? a. Assessing the IV site before giving the drug b. Obtaining a programmable (smart) IV pump c. Removing the IV bag from the brown plastic cover d. Taking and recording a baseline set of vital signs

ANS: C Nitroprusside degrades in the presence of light, so it must be protected by leaving it in the original brown plastic bag when infusing. The other actions are correct, although a smart pump is not necessarily required if the facility does not have them available. The drug must be administered via an IV pump, although the programmable pump is preferred for safety.

A client presents to the emergency department with an acute myocardial infarction (MI) at 1500 (3:00 PM). The facility has 24-hour catheterization laboratory abilities. To meet The Joint Commissions Core Measures set, by what time should the client have a percutaneous coronary intervention performed? a. 1530 (3:30 PM) b. 1600 (4:00 PM) c. 1630 (4:30 PM) d. 1700 (5:00 PM)

ANS: C The Joint Commissions Core Measures set for MI includes percutaneous coronary intervention within 90 minutes of diagnosis of myocardial infarction. Therefore, the client should have a percutaneous coronary intervention performed no later than 1630 (4:30 PM).

A client is on a ventilator and is sedated. What care may the nurse delegate to the unlicensed assistive personnel (UAP)? A. Assess the client for sedation needs. B. Get family permission for restraints. C. Provide frequent oral care per protocol. D. Use nonverbal pain assessment tools.

ANS: C The client on mechanical ventilation needs frequent oral care, which can be delegated to the UAP. The other actions fall within the scope of practice of the nurse.

A nurse cares for a client who tests positive for alpha1-antitrypsin (AAT) deficiency. The client asks, What does this mean? How should the nurse respond? a. Your children will be at high risk for the development of chronic obstructive pulmonary disease .b. I will contact a genetic counselor to discuss your condition. c. Your risk for chronic obstructive pulmonary disease is higher, especially if you smoke. d. This is a recessive gene and should have no impact on your health.

ANS: C The gene for AAT is a recessive gene. Clients with only one allele produce enough AAT to prevent chronic obstructive pulmonary disease (COPD) unless the client smokes. A client with two alleles is at high risk for COPD even if not exposed to smoke or other irritants. The client is a carrier, and children may or may not be at high risk depending on the partners AAT levels. Contacting a genetic counselor may be helpful but does not address the clients current question.

A nurse performs an admission assessment on a 75-year-old client with multiple chronic diseases. The clients blood pressure is 135/75 mm Hg and oxygen saturation is 94% on 2 liters per nasal cannula. The nurse assesses the clients rhythm on the cardiac monitor and observes the reading shown below: Which action should the nurse take first? a. Begin external temporary pacing. b. Assess peripheral pulse strength. c. Ask the client what medications he or she takes. d. Administer 1 mg of atropine.

ANS: C This client is stable and therefore does not require any intervention except to determine the cause of the bradycardia. Bradycardia is often caused by medications. Clients who have multiple chronic diseases are often on multiple medications that can interact with each other. The nurse should assess the clients current medications first.

A client is receiving an infusion of tissue plasminogen activator (t-PA). The nurse assesses the client to be disoriented to person, place, and time. What action by the nurse is best? a. Assess the clients pupillary responses. b. Request a neurologic consultation. c. Stop the infusion and call the provider. d. Take and document a full set of vital signs.

ANS: C A change in neurologic status in a client receiving t-PA could indicate intracranial hemorrhage. The nurse should stop the infusion and notify the provider immediately. A full assessment, including pupillary responses and vital signs, occurs next. The nurse may or may not need to call a neurologist.

A telemetry nurse assesses a client who has a heart rate of 35 beats/min on the cardiac monitor. Which assessment would the nurse complete next? a. Pulmonary auscultation b. Pulse strength and amplitude c. Level of consciousness d. Mobility and gait stability

ANS: C A heart rate of 40 beats/min or less could have hemodynamic consequences. The client is at risk for inadequate cerebral perfusion. The nurse would assess for level of consciousness, dizziness, confusion, syncope, chest pain, shortness of breath. Although the other assessments would be completed, the nurse would assess the client's neurologic status next.

A telemetry nurse assesses a client with third-degree heart block who has wide QRS complexes and a heart rate of 35 beats/min on the cardiac monitor. Which assessment should the nurse complete next? a. Pulmonary auscultation b. Pulse strength and amplitude c. Level of consciousness d. Mobility and gait stability

ANS: C A heart rate of 40 beats/min or less with widened QRS complexes could have hemodynamic consequences. The client is at risk for inadequate cerebral perfusion. The nurse should assess for level of consciousness, light- headedness, confusion, syncope, and seizure activity. Although the other assessments should be completed, the clients level of consciousness is the priority.

A client has a pulmonary embolism and is started on oxygen. The student nurse asks why the clients oxygen saturation has not significantly improved. What response by the nurse is best? a. Breathing so rapidly interferes with oxygenation. b. Maybe the client has respiratory distress syndrome. c. The blood clot interferes with perfusion in the lungs. d. The client needs immediate intubation and mechanical ventilation.

ANS: C A large blood clot in the lungs will significantly impair gas exchange and oxygenation. Unless the clot is dissolved, this process will continue unabated. Hyperventilation can interfere with oxygenation by shallow breathing, but there is no evidence that the client is hyperventilating, and this is also not the most precise physiologic answer. Respiratory distress syndrome can occur, but this is not as likely. The client may need to be mechanically ventilated, but without concrete data on FiO2 and SaO2, the nurse cannot make that judgment.

A nurse assesses a client who is recovering after a left-sided cardiac catheterization. Which assessment finding requires immediate intervention? a. Urinary output less than intake b. Bruising at the insertion site c. Slurred speech and confusion d. Discomfort in the left leg

ANS: C A left-sided cardiac catheterization specifically increases the risk for a cerebral vascular accident. A change in neurologic status needs to be acted on immediately. Discomfort and bruising are expected at the site. If intake decreases, a client can become dehydrated because of dye excretion. The second intervention would be to increase the clients fluid status. Neurologic changes would take priority.

A nurse uses the rule of nines to assess a client with burn injuries to the entire back region and left arm. How should the nurse document the percentage of the clients body that sustained burns? a. 9% b. 8% c. 27% d. 36%

ANS: C According to the rule of nines, the posterior trunk, anterior trunk, and legs each make up 18% of the total body surface. The head, neck, and arms each make up 9% of total body surface, and the perineum makes up 1%. In this case, the client received burns to the back (18%) and one arm (9%), totaling 27% of the body.

After teaching a client who is recovering from a heart transplant to change positions slowly, the client asks, Why is this important? How should the nurse respond? a. Rapid position changes can create shear and friction forces, which can tear out your internal vascular sutures. b. Your new vascular connections are more sensitive to position changes, leading to increased intravascular pressure and dizziness. c. Your new heart is not connected to the nervous system and is unable to respond to decreases in blood pressure caused by position changes. d. While your heart is recovering, blood flow is diverted away from the brain, increasing the risk for stroke when you stand up.

ANS: C Because the new heart is denervated, the baroreceptor and other mechanisms that compensate for blood pressure drops caused by position changes do not function. This allows orthostatic hypotension to persist in the postoperative period. The other options are false statements and do not correctly address the clients question.

A nurse administers prescribed adenosine to a client. Which response would the nurse assess for as the expected therapeutic response? a. Decreased intraocular pressure b. Increased heart rate c. Short period of asystole d. Hypertensive crisis

ANS: C Clients usually respond to adenosine with a short period of asystole, bradycardia with long pauses, nausea, or vomiting. Adenosine has no impact on intraocular pressure nor does it cause increased heart rate or hypertensive crisis.

A nurse administers prescribed adenosine (Adenocard) to a client. Which response should the nurse assess for as the expected therapeutic response? a. Decreased intraocular pressure b. Increased heart rate c. Short period of asystole d. Hypertensive crisis

ANS: C Clients usually respond to adenosine with a short period of asystole, bradycardia, hypotension, dyspnea, and chest pain. Adenosine has no conclusive impact on intraocular pressure.

A nurse cares for a client who is recovering from a myocardial infarction. The client states, I will need to stop eating so much chili to keep that indigestion pain from returning. How should the nurse respond? a. Chili is high in fat and calories; it would be a good idea to stop eating it. b. The provider has prescribed an antacid for you to take every morning. c. What do you understand about what happened to you? d. When did you start experiencing this indigestion?

ANS: C Clients who experience myocardial infarction often respond with denial, which is a defense mechanism. The nurse should ask the client what he or she thinks happened, or what the illness means to him or her. The other responses do not address the clients misconception about recent pain and the cause of that pain.

A nurse cares for a client with atrial fibrillation who reports fatigue when completing activities of daily living. What intervention would the nurse implement to address this client's concerns? a. Administer oxygen therapy at 2 L per nasal cannula. b. Provide the client with a sleeping pill to stimulate rest. c. Schedule periods of exercise and rest during the day. d. Ask assistive personnel (AP) to help bathe the client.

ANS: C Clients who have atrial fibrillation are at risk for decreased cardiac output and fatigue when completing activities of daily living. The nurse would schedule periods of exercise and rest during the day to decrease fatigue. The other interventions will not assist the client with performing self-care activities and there is no indication for oxygen.

The nurse asks a client who has experienced ventricular dysrhythmias about substance abuse. The client asks, "Why do you want to know if I use cocaine?" What is the nurse's best response? a. "Substance abuse puts clients at risk for many health issues." b. "The hospital requires that I ask you about cocaine use." c. "Clients who use cocaine are at risk for fatal dysrhythmias." d. "We can provide services for cessation of substance abuse."

ANS: C Clients who use cocaine or illicit inhalants are particularly at risk for potentially fatal dysrhythmias. The other responses do not adequately address the client's question.

A nurse is teaching a client who has cystic fibrosis (CF). Which statement should the nurse include in this clients teaching? a. Take an antibiotic each day. b. Contact your provider to obtain genetic screening. c. Eat a well-balanced, nutritious diet. d. Plan to exercise for 30 minutes every day.

ANS: C Clients with CF often are malnourished due to vitamin deficiency and pancreatic malfunction. Maintaining nutrition is essential. Daily antibiotics and daily exercise are not essential actions. Genetic screening would not help the client manage CF better.

A nurse assesses a client in an outpatient clinic. Which statement alerts the nurse to the possibility of left- sided heart failure? a. I have been drinking more water than usual. b. I am awakened by the need to urinate at night. c. I must stop halfway up the stairs to catch my breath. d. I have experienced blurred vision on several occasions.

ANS: C Clients with left-sided heart failure report weakness or fatigue while performing normal activities of daily living, as well as difficulty breathing, or catching their breath. This occurs as fluid moves into the alveoli. Nocturia is often seen with right-sided heart failure. Thirst and blurred vision are not related to heart failure.

A nurse cares for a client with chronic obstructive pulmonary disease (COPD) who appears thin and disheveled. Which question should the nurse ask first? a. Do you have a strong support system? b. What do you understand about your disease? c. Do you experience shortness of breath with basic activities? d. What medications are you prescribed to take each day?

ANS: C Clients with severe COPD may not be able to perform daily activities, including bathing and eating, because of excessive shortness of breath. The nurse should ask the client if shortness of breath is interfering with basic activities. Although the nurse should know about the clients support systems, current knowledge, and medications, these questions do not address the clients appearance.

A nurse is assessing an obese client in the clinic for follow-up after an episode of deep vein thrombosis. The client has lost 20 pounds since the last visit. What action by the nurse is best? a. Ask if the weight loss was intended. b. Encourage a high-protein, high-fiber diet. c. Measure for new compression stockings. d. Review a 3-day food recall diary.

ANS: C Compression stockings must fit correctly in order to work. After losing a significant amount of weight, the client should be re-measured and new stockings ordered if needed. The other options are appropriate, but not the most important.

A nurse cares for a female client who has a family history of cystic fibrosis. The client asks, Will my children have cystic fibrosis? How should the nurse respond? a. Since many of your family members are carriers, your children will also be carriers of the gene. b. Cystic fibrosis is an autosomal recessive disorder. If you are a carrier, your children will have the disorder. c. Since you have a family history of cystic fibrosis, I would encourage you and your partner to be tested. d. Cystic fibrosis is caused by a protein that controls the movement of chloride. Adjusting your diet will decrease the spread of this disorder.

ANS: C Cystic fibrosis is an autosomal recessive disorder in which both gene alleles must be mutated for the disorder to be expressed. The nurse should encourage both the client and partner to be tested for the abnormal gene. The other statements are not true.

A nurse is assisting the health care provider who is intubating a client. The provider has been attempting to intubate for 40 seconds. What action by the nurse takes priority? a. Ensure the client has adequate sedation. b. Find another provider to intubate. c. Interrupt the procedure to give oxygen. d. Monitor the clients oxygen saturation.

ANS: C Each intubation attempt should not exceed 30 seconds (15 is preferable) as it causes hypoxia. The nurse should interrupt the intubation attempt and give the client oxygen. The nurse should also have adequate sedation during the procedure and monitor the clients oxygen saturation, but these do not take priority. Finding another provider is not appropriate at this time.

A client is admitted with a pulmonary embolism (PE). The client is young, healthy, and active and has no known risk factors for PE. What action by the nurse is most appropriate? A. Encourage the client to walk 5 minutes each hour. B. Refer the client to smoking cessation classes. C. Teach the client about factor V Leiden testing. D. Tell the client that sometimes no cause for disease is found.

ANS: C Factor V Leiden is an inherited thrombophilia that can lead to abnormal clotting events, including PE. A client with no known risk factors for this disorder should be referred for testing. Encouraging the client to walk is healthy, but is not related to the development of a PE in this case, nor is smoking. Although there are cases of disease where no cause is ever found, this assumption is premature.

A client has an intra-arterial blood pressure monitoring line. The nurse notes bright red blood on the clients sheets. What action should the nurse perform first? a. Assess the insertion site. b. Change the clients sheets. c. Put on a pair of gloves. d. Assess blood pressure.

ANS: C For the nurses safety, he or she should put on a pair of gloves to prevent blood exposure. The other actions are appropriate as well, but first the nurse must don a pair of gloves.

The nurse teaches burn prevention to a community group. Which statement by a member of the group should cause the nurse the greatest concern? a.I get my chimney swept every other year. b.My hot water heater is set at 120 degrees. c.Sometimes I wake up at night and smoke. d.I use a space heater when it gets below zero.

ANS: C House fires are a common occurrence and often lead to serious injury or death. The nurse should be most concerned about a person who wakes up at night and smokes. The nurse needs to question this person about whether he or she gets out of bed to do so, or if this person stays in bed, which could lead to falling back asleep with a lighted cigarette. Although it is recommended to have chimneys swept every year, skipping a year does not pose as much danger as smoking in bed, particularly if the person does not burn wood frequently. Water heaters should be set below 140 F. Space heaters should be used with caution, and the nurse may want to ensure that the person does not allow it to get near clothing or bedding.

A nurse assesses an older adult client who is experiencing a myocardial infarction. Which clinical manifestation should the nurse expect? a. Excruciating pain on inspiration b. Left lateral chest wall pain c. Disorientation and confusion d. Numbness and tingling of the arm

ANS: C In older adults, disorientation or confusion may be the major manifestation of myocardial infarction caused by poor cardiac output. Pain manifestations and numbness and tingling of the arm could also be related to the myocardial infarction. However, the nurse should be more concerned about the new onset of disorientation or confusion caused by decreased perfusion.

A nurse assesses a client who has a burn injury. Which statement indicates the client has a positive perspective of his or her appearance? a. I will allow my spouse to change my dressings. b. I want to have surgical reconstruction. c. I will bathe and dress before breakfast. d. I have secured the pressure dressings as ordered.

ANS: C Indicators that the client with a burn injury has a positive perception of his or her appearance include a willingness to touch the affected body part. Self-care activities such as morning care foster feelings of self- worth, which are closely linked to body image. Allowing others to change the dressing and discussing future reconstruction would not indicate a positive perception of appearance. Wearing the dressing will assist in decreasing complications but will not enhance self-perception.

A nurse cares for a client with a burn injury who presents with drooling and difficulty swallowing. Which action should the nurse take first? a. Assess the level of consciousness and pupillary reactions. b. Ascertain the time food or liquid was last consumed. c. Auscultate breath sounds over the trachea and bronchi. d. Measure abdominal girth and auscultate bowel sounds.

ANS: C Inhalation injuries are present in 7% of clients admitted to burn centers. Drooling and difficulty swallowing can mean that the client is about to lose his or her airway because of this injury. Absence of breath sounds over the trachea and bronchi indicates impending airway obstruction and demands immediate intubation. Knowing the level of consciousness is important in assessing oxygenation to the brain. Ascertaining the time of last food intake is important in case intubation is necessary (the nurse will be more alert for signs of aspiration). However, assessing for air exchange is the most important intervention at this time. Measuring abdominal girth is not relevant in this situation.

A nurse assesses a client 2 hours after a cardiac angiography via the left femoral artery. The nurse notes that the left pedal pulse is weak. Which action should the nurse take? a. Elevate the leg and apply a sandbag to the entrance site. b. Increase the flow rate of intravenous fluids. c. Assess the color and temperature of the left leg. d. Document the finding as left pedal pulse of +1/4.

ANS: C Loss of a pulse distal to an angiography entry site is serious, indicating a possible arterial obstruction. The pulse may be faint because of edema. The left pulse should be compared with the right, and pulses should be compared with previous assessments, especially before the procedure. Assessing color (pale, cyanosis) and temperature (cool, cold) will identify a decrease in circulation. Once all peripheral and vascular assessment data are acquired, the primary health care provider should be notified. Simply documenting the findings is inappropriate. The leg should be positioned below the level of the heart or dangling to increase blood flow to the distal portion of the leg. Increasing intravenous fluids will not address the clients problem.

A nurse cares for a client who has facial burns. The client asks, Will I ever look the same? How should the nurse respond? a. With reconstructive surgery, you can look the same. b. We can remove the scars with the use of a pressure dressing. c. You will not look exactly the same but cosmetic surgery will help. d. You shouldnt start worrying about your appearance right now.

ANS: C Many clients have unrealistic expectations of reconstructive surgery and envision an appearance identical or equal in quality to the preburn state. The nurse should provide accurate information that includes something to hope for. Pressure dressings prevent further scarring; they cannot remove scars. The client and the family should be taught the expected cosmetic outcomes.

A nurse cares for a client with chronic obstructive pulmonary disease (COPD). The client states that he no longer enjoys going out with his friends. How should the nurse respond? a. There are a variety of support groups for people who have COPD. b. I will ask your provider to prescribe you with an antianxiety agent. c. Share any thoughts and feelings that cause you to limit social activities. d. Friends can be a good support system for clients with chronic disorders.

ANS: C Many clients with moderate to severe COPD become socially isolated because they are embarrassed by frequent coughing and mucus production. They also can experience fatigue, which limits their activities. The nurse needs to encourage the client to verbalize thoughts and feelings so that appropriate interventions can be selected. Joining a support group would not decrease feelings of social isolation if the client does not verbalize feelings. Antianxiety agents will not help the client with social isolation. Encouraging a client to participate in activities without verbalizing concerns also would not be an effective strategy for decreasing social isolation.

A nurse is caring for a client who has sleep apnea and is prescribed modafinil (Provigil). The client asks, How will this medication help me? How should the nurse respond? a. This medication will treat your sleep apnea. b. This sedative will help you to sleep at night. c. This medication will promote daytime wakefulness. d. This analgesic will increase comfort while you sleep.

ANS: C Modafinil is helpful for clients who have narcolepsy (uncontrollable daytime sleep) related to sleep apnea. This medication promotes daytime wakefulness.

Vital Signs Heart rate: 110 beats/min Blood pressure: 112/68 mm Hg Respiratory rate: 20 Laboratory Results Red blood cell count: 5,000,000/mm3 White blood cell count: 10,000/mm3 Platelet count: 200,000/mm3 Wound Assessment Left chest burn wound, 3 cm 2.5 cm 0.5 cm, wound bed pale, surrounding tissues with edema breaths/min present Oxygen saturation: 94% Pain: 3/10Based on the documented data, which action should the nurse take next? a. Assess the clients skin for signs of adequate perfusion. b. Calculate intake and output ratio for the last 24 hours. c. Prepare to obtain blood and wound cultures. d. Place the client in an isolation room.

ANS: C Older clients have a decreased immune response, so they may not exhibit signs that their immune system is actively fighting an infection, such as fever or an increased white blood cell count. They also are at higher risk for sepsis arising from a localized wound infection. The burn wound shows signs of local infection, so the nurse should assess for this and for systemic infection before the client manifests sepsis. Placing the client in an isolation room, calculating intake and output, and assessing the clients skin should all be implemented but these actions do not take priority over determining whether the client has an infection.

A nurse assesses an older adult client who has multiple chronic diseases. The clients heart rate is 48 beats/min. Which action should the nurse take first? a. Document the finding in the chart. b. Initiate external pacing. c. Assess the clients medications. d. Administer 1 mg of atropine.

ANS: C Pacemaker cells in the conduction system decrease in number as a person ages, resulting in bradycardia. The nurse should check the medication reconciliation for medications that might cause such a drop in heart rate, then should inform the health care provider. Documentation is important, but it is not the priority action. The heart rate is not low enough for atropine or an external pacemaker to be needed.

A client had an acute myocardial infarction. What assessment finding indicates to the nurse that a significant complication has occurred? a. Blood pressure that is 20 mm Hg below baseline b. Oxygen saturation of 94% on room air c. Poor peripheral pulses and cool skin d. Urine output of 1.2 mL/kg/hr for 4 hours

ANS: C Poor peripheral pulses and cool skin may be signs of impending cardiogenic shock and should be reported immediately. A blood pressure drop of 20 mm Hg is not worrisome. An oxygen saturation of 94% is just slightly below normal. A urine output of 1.2 mL/kg/hr for 4 hours is normal.

After teaching a client who is prescribed salmeterol (Serevent), the nurse assesses the clients understanding. Which statement by the client indicates a need for additional teaching? a. I will be certain to shake the inhaler well before I use it. b. It may take a while before I notice a change in my asthma. c. I will use the drug when I have an asthma attack. d. I will be careful not to let the drug escape out of my nose and mouth.

ANS: C Salmeterol is designed to prevent an asthma attack; it does not relieve or reverse symptoms. Salmeterol has a slow onset of action; therefore, it should not be used as a rescue drug. The drug must be shaken well because it has a tendency to separate easily. Poor technique on the clients part allows the drug to escape through the nose and mouth.

A nurse cares for a client who has developed esophagitis after undergoing radiation therapy for lung cancer. Which diet selection should the nurse provide for this client? a. Spaghetti with meat sauce, ice cream b. Chicken soup, grilled cheese sandwich c. Omelet, soft whole wheat bread d. Pasta salad, custard, orange juice

ANS: C Side effects of radiation therapy may include inflammation of the esophagus. Clients should be taught that bland, soft, high-calorie foods are best, along with liquid nutritional supplements. Tomato sauce may prove too spicy for a client with esophagitis. A grilled cheese sandwich is too difficult to swallow with this condition, and orange juice and other foods with citric acid are too caustic.

A nurse teaches a client being treated for a full-thickness burn. Which statement should the nurse include in this clients discharge teaching? a. You should change the batteries in your smoke detector once a year. b. Join a program that assists burn clients to reintegration into the community. c. I will demonstrate how to change your wound dressing for you and your family. d. Let me tell you about the many options available to you for reconstructive surgery.

ANS: C Teaching clients and family members to perform care tasks such as dressing changes is critical for the progressive goal toward independence for the client. All of the other options are important in the rehabilitation stage. However, dressing changes have priority.

Admission Notes: 36-year-old female with Wound Assessment Bilateral leg burns present bilateral leg burns NKDA Health history of asthma and seasonal allergies with a white and leather- like appearance. No blisters or bleeding present. Client rates pain 2/10 on a scale of 0- 10. Based on the data provided, how should the nurse categorize this clients injuries? a. Partial-thickness deep b. Partial-thickness superficial c. Full thickness d. Superficial

ANS: C The characteristics of the clients wounds meet the criteria for a full-thickness injury: color that is black, brown, yellow, white, or red; no blisters; minimal pain; and firm and inelastic outer layer. Partial-thickness superficial burns appear pink to red and are painful. Partial-thickness deep burns are deep red to white and painful. Superficial burns are pink to red and are also painful.

A nurse assesses clients on the medical-surgical unit. Which client is at greatest risk for development of obstructive sleep apnea? a. A 26-year-old woman who is 8 months pregnant b. A 42-year-old man with gastroesophageal reflux disease c. A 55-year-old woman who is 50 pounds overweight d. A 73-year-old man with type 2 diabetes mellitus

ANS: C The client at highest risk would be the one who is extremely overweight. None of the other clients have risk factors for sleep apnea.

A nurse cares for a client who is scheduled for a total laryngectomy. Which action should the nurse take prior to surgery? a. Assess airway patency, breathing, and circulation. b. Administer prescribed intravenous pain medication. c. Assist the client to choose a communication method. d. Ambulate the client in the hallway to assess gait.

ANS: C The client will not be able to speak after surgery. The nurse should assist the client to choose a communication method that he or she would like to use after surgery. Assessing the clients airway and administering IV pain medication are done after the procedure. Although ambulation promotes health and decreases the complications of any surgery, this clients gait should not be impacted by a total laryngectomy and therefore is not a priority.

A nurse assesses clients on a medical-surgical unit. Which client should the nurse identify as having the greatest risk for cardiovascular disease? a. An 86-year-old man with a history of asthma b. A 32-year-old Asian-American man with colorectal cancer c. A 45-year-old American Indian woman with diabetes mellitus d. A 53-year-old postmenopausal woman who is on hormone therapy.

ANS: C The incidence of coronary artery disease and hypertension is higher in American Indians than in whites or Asian Americans. Diabetes mellitus increases the risk for hypertension and coronary artery disease in people of any race or ethnicity. Asthma, colorectal cancer, and hormone therapy do not increase risk for cardiovascular disease

A nurse prepares a client for coronary artery bypass graft surgery. The client states, I am afraid I might die. How should the nurse respond? a. This is a routine surgery and the risk of death is very low. b. Would you like to speak with a chaplain prior to surgery? c. Tell me more about your concerns about the surgery. d. What support systems do you have to assist you?

ANS: C The nurse should discuss the clients feelings and concerns related to the surgery. The nurse should not provide false hope or push the clients concerns off on the chaplain. The nurse should address support systems after addressing the clients current issue.

The nurse instructs a client on how to correctly use an inhaler with a spacer. In which order should these steps occur? 1. Press down firmly on the canister to release one dose of medication. 2. Breathe in slowly and deeply. 3. Shake the whole unit vigorously three or four times. 4. Insert the mouthpiece of the inhaler into the nonmouthpiece end of the spacer. 5. Place the mouthpiece into your mouth, over the tongue, and seal your lips tightly around the mouthpiece. 6. Remove the mouthpiece from your mouth, keep your lips closed, and hold your breath for at least 10 seconds. a. 2, 3, 4, 5, 6, 1 b. 3, 4, 5, 1, 6, 2 c. 4, 3, 5, 1, 2, 6 d. 5, 3, 6, 1, 2, 4

ANS: C The proper order for correctly using an inhaler with a spacer is as follows. Insert the mouthpiece of the inhaler into the nonmouthpiece end of the spacer. Shake the whole unit vigorously three or four times. Place the mouthpiece into the mouth, over the tongue, and seal the lips tightly around it. Press down firmly on the canister of the inhaler to release one dose of medication into the spacer. Breathe in slowly and deeply. Remove the mouthpiece from the mouth, and, keeping the lips closed, hold the breath for at least 10 seconds. Then breathe out slowly. Wait at least 1 minute between puffs.

A nurse reviews the laboratory results for a client who was burned 24 hours ago. Which laboratory result should the nurse report to the health care provider immediately? a. Arterial pH: 7.32 b. Hematocrit: 52% c. Serum potassium: 6.5 mEq/L d. Serum sodium: 131 mEq/L

ANS: C The serum potassium level is changed to the degree that serious life-threatening responses could result. With such a rapid rise in potassium level, the client is at high risk for experiencing severe cardiac dysrhythmias and death. All the other findings are abnormal but do not show the same degree of severity; they would be expected in the emergent phase after a burn injury.

A nurse caring for a client notes the following assessments: white blood cell count 3800/mm3, blood glucose level 198 mg/dL, and temperature 96.2 F (35.6 C). What action by the nurse takes priority? a. Document the findings in the clients chart. b. Give the client warmed blankets for comfort. c. Notify the health care provider immediately. d. Prepare to administer insulin per sliding scale.

ANS: C This client has several indicators of sepsis with systemic inflammatory response. The nurse should notify the health care provider immediately. Documentation needs to be thorough but does not take priority. The client may appreciate warm blankets, but comfort measures do not take priority. The client may or may not need insulin.

A nurse prepares to administer intravenous cimetidine (Tagamet) to a client who has a new burn injury. The client asks, Why am I taking this medication? How should the nurse respond? a. Tagamet stimulates intestinal movement so you can eat more. b. It improves fluid retention, which helps prevent hypovolemic shock. c. It helps prevent stomach ulcers, which are common after burns. d. Tagamet protects the kidney from damage caused by dehydration.

ANS: C Ulcerative gastrointestinal disease (Curlings ulcer) may develop within 24 hours after a severe burn as a result of increased hydrochloric acid production and a decreased mucosal barrier. This process occurs because of the sympathetic nervous system stress response. Cimetidine is a histamine2 blocker and inhibits the production and release of hydrochloric acid. Cimetidine does not affect intestinal movement and does not prevent hypovolemic shock or kidney damage.

An intubated clients oxygen saturation has dropped to 88%. What action by the nurse takes priority? a. Determine if the tube is kinked. b. Ensure all connections are patent. c. Listen to the clients lung sounds. d. Suction the endotracheal tube.

ANS: C When an intubated client shows signs of hypoxia, check for DOPE: displaced tube (most common cause), obstruction (often by secretions), pneumothorax, and equipment problems. The nurse listens for equal, bilateral breath sounds first to determine if the endotracheal tube is still correctly placed. If this assessment is normal, the nurse would follow the mnemonic and assess the patency of the tube and connections and perform suction.

A nurse assesses a client who is scheduled for a cardiac catheterization. Which assessment should the nurse complete prior to this procedure? a. Clients level of anxiety b. Ability to turn self in bed c. Cardiac rhythm and heart rate d. Allergies to iodine-based agent.

ANS: D Before the procedure, the nurse should ascertain whether the client has an allergy to iodine-containing preparations, such as seafood or local anesthetics. The contrast medium used during the procedure is iodine based. This allergy can cause a life-threatening reaction, so it is a high priority. Second, it is important for the nurse to assess anxiety, mobility, and baseline cardiac status.

An older client with peripheral vascular disease (PVD) is explaining the daily foot care regimen to the family practice clinic nurse. What statement by the client may indicate a barrier to proper foot care? a. I nearly always wear comfy sweatpants and house shoes. b. Im glad I get energy assistance so my house isnt so cold. c. My daughter makes sure I have plenty of lotion for my feet .d. My hands shake when I try to do things requiring coordination.

ANS: D Clients with PVD need to pay special attention to their feet. Toenails need to be kept short and cut straight across. The client whose hands shake may cause injury when trimming toenails. The nurse should refer this client to a podiatrist. Comfy sweatpants and house shoes are generally loose and not restrictive, which is important for clients with PVD. Keeping the house at a comfortable temperature makes it less likely the client will use alternative heat sources, such as heating pads, to stay warm. The client should keep the feet moist and soft with lotion.

A nurse evaluates the following arterial blood gas and vital sign results for a client with chronic obstructive pulmonary disease (COPD): Arterial Blood Gas Results Vital Signs: pH = 7.32 PaCO2 = 62 mm Hg PaO2 = 46 mm Hg HCO3 = 28 mEq/L Heart rate = 110 beats/min Respiratory rate = 12 breaths/min Blood pressure = 145/65 mm HgOxygen saturation = 76% Which action should the nurse take first? a. Administer a short-acting beta2 agonist inhaler. b. Document the findings as normal for a client with COPD. c. Teach the client diaphragmatic breathing techniques. d. Initiate oxygenation therapy to increase saturation to 92%.

ANS: D Oxygen should be administered to a client who is hypoxic even if the client has COPD and is a carbon dioxide retainer. The other interventions do not address the clients hypoxia, which is the priority

A nurse assesses a clients electrocardiogram (ECG) and observes the reading shown below: How should the nurse document this clients ECG strip? a. Ventricular tachycardia b. Ventricular fibrillation c. Sinus rhythm with premature atrial contractions (PACs) d. Sinus rhythm with premature ventricular contractions (PVCs)

ANS: D Sinus rhythm with PVCs has an underlying regular sinus rhythm with ventricular depolarization that sometimes precede atrial depolarization. Ventricular tachycardia and ventricular fibrillation rhythms would not have sinus beats present. Premature atrial contractions are atrial contractions initiated from another region of the atria before the sinus node initiates atrial depolarization.

A student nurse is assessing the peripheral vascular system of an older adult. What action by the student would cause the faculty member to intervene? a. Assessing blood pressure in both upper extremities b. Auscultating the carotid arteries for any bruits c. Classifying capillary refill of 4 seconds as normal d. Palpating both carotid arteries at the same time

ANS: D The student should not compress both carotid arteries at the same time to avoid brain ischemia. Blood pressure should be taken and compared in both arms. Prolonged capillary refill is considered to be greater than 5 seconds in an older adult, so classifying refill of 4 seconds as normal would not require intervention. Bruits should be auscultated.

The nurse is preparing to change a clients sternal dressing. What action by the nurse is most important? a. Assess vital signs. b. Don a mask and gown. c. Gather needed supplies. d. Perform hand hygiene.

ANS: D To prevent a sternal wound infection, the nurse washes hands or performs hand hygiene as a priority. Vital signs do not necessarily need to be assessed beforehand. A mask and gown are not needed. The nurse should gather needed supplies, but this is not the priority.

The nurse is reviewing the lipid panel of a male client who has atherosclerosis. Which finding is most concerning? a. Cholesterol: 126 mg/dL b. High-density lipoprotein cholesterol (HDL-C): 48 mg/dL c. Low-density lipoprotein cholesterol (LDL-C): 122 mg/dL d. Triglycerides: 198 mg/dL

ANS: D Triglycerides in men should be below 160 mg/dL. The other values are appropriate for adult males.

A nurse cares for a client who had a chest tube placed 6 hours ago and refuses to take deep breaths because of the pain. Which action should the nurse take? a. Ambulate the client in the hallway to promote deep breathing. b. Auscultate the clients anterior and posterior lung fields. c. Encourage the client to take shallow breaths to help with the pain. d. Administer pain medication and encourage the client to take deep breaths.

ANS: D A chest tube is placed in the pleural space and may be uncomfortable for a client. The nurse should provide pain medication to minimize discomfort and encourage the client to take deep breaths.

A nursing student is caring for a client with an abdominal aneurysm. What action by the student requires the registered nurse to intervene? a. Assesses the client for back pain b. Auscultates over abdominal bruit c. Measures the abdominal girth d. Palpates the abdomen in four quadrants

ANS: D Abdominal aneurysms should never be palpated as this increases the risk of rupture. The registered nurse should intervene when the student attempts to do this. The other actions are appropriate.

An emergency department nurse triages clients who present with chest discomfort. Which client should the nurse plan to assess first? a. A 42-year-old female who describes her pain as a dull ache with numbness in her fingers b. A 49-year-old male who reports moderate pain that is worse on inspiration c. A 53-year-old female who reports substernal pain that radiates to her abdomen d. A 58-year-old male who describes his pain as intense stabbing that spreads across his chest

ANS: D All clients who have chest pain should be assessed more thoroughly. To determine which client should be seen first, the nurse must understand common differences in pain descriptions. Intense stabbing, vise-like substernal pain that spreads through the clients chest, arms, jaw, back, or neck is indicative of a myocardial infarction. The nurse should plan to see this client first to prevent cardiac cell death. A dull ache with numbness in the fingers is consistent with anxiety. Pain that gets worse with inspiration is usually related to a pleuropulmonary problem. Pain that spreads to the abdomen is often associated with an esophageal-gastric problem, especially when this pain is experienced by a male client. Female clients may experience abdominal discomfort with a myocardial event. Although clients with anxiety, pleuropulmonary, and esophageal-gastric problems should be seen, they are not a higher priority than myocardial infarction.

A nurse assesses a client after administering a prescribed beta blocker. Which assessment should the nurse expect to find? a. Blood pressure increased from 98/42 mm Hg to 132/60 mm Hg b. Respiratory rate decreased from 25 breaths/min to 14 breaths/min c. Oxygen saturation increased from 88% to 96% d. Pulse decreased from 100 beats/min to 80 beats/min

ANS: D Beta blockers block the stimulation of beta1-adrenergic receptors. They block the sympathetic (fight-or-flight) response and decrease the heart rate (HR). The beta blocker will decrease HR and blood pressure, increasing ventricular filling time. It usually does not have effects on beta2-adrenergic receptor sites. Cardiac output will drop because of decreased HR.

A nurse assesses a client who reports waking up feeling very tired, even after 8 hours of good sleep. Which action should the nurse take first? a. Contact the provider for a prescription for sleep medication. b. Tell the client not to drink beverages with caffeine before bed. c. Educate the client to sleep upright in a reclining chair. d. Ask the client if he or she has ever been evaluated for sleep apnea.

ANS: D Clients are usually unaware that they have sleep apnea, but it should be suspected in people who have persistent daytime sleepiness and report waking up tired. Causes of the problem should be assessed before the client is offered suggestions for treatment.

A nurse teaches a client who has a history of heart failure. Which statement should the nurse include in this clients discharge teaching? a. Avoid drinking more than 3 quarts of liquids each day. b. Eat six small meals daily instead of three larger meals. c. When you feel short of breath, take an additional diuretic. d. Weigh yourself daily while wearing the same amount of clothing.

ANS: D Clients with heart failure are instructed to weigh themselves daily to detect worsening heart failure early, and thus avoid complications. Other signs of worsening heart failure include increasing dyspnea, exercise intolerance, cold symptoms, and nocturia. Fluid overload increases symptoms of heart failure. The client should be taught to eat a heart-healthy diet, balance intake and output to prevent dehydration and overload, and take medications as prescribed. The most important discharge teaching is daily weights as this provides the best data related to fluid retention.

A nurse assesses bilateral wheezes in a client with burn injuries inside the mouth. Four hours later the wheezing is no longer heard. Which action should the nurse take? a. Document the findings and reassess in 1 hour. b. Loosen any constrictive dressings on the chest. c. Raise the head of the bed to a semi-Fowlers position. d. Gather appropriate equipment and prepare for an emergency airway.

ANS: D Clients with severe inhalation injuries may sustain such progressive obstruction that they may lose effective movement of air. When this occurs, wheezing is no longer heard, and neither are breath sounds. These clients can lose their airways very quickly, so prompt action is needed. The client requires establishment of an emergency airway. Swelling usually precludes intubation. The other options do not address this emergency situation.

A nurse teaches a client who is prescribed digoxin (Lanoxin) therapy. Which statement should the nurse include in this clients teaching? a. Avoid taking aspirin or aspirin-containing products. b. Increase your intake of foods that are high in potassium. c. Hold this medication if your pulse rate is below 80 beats/min. d. Do not take this medication within 1 hour of taking an antacid.

ANS: D Gastrointestinal absorption of digoxin is erratic. Many medications, especially antacids, interfere with its absorption. Clients are taught to hold their digoxin for bradycardia; a heart rate of 80 beats/min is too high for this cutoff. Potassium and aspirin have no impact on digoxin absorption, nor do these statements decrease complications of digoxin therapy.

The registered nurse assigns a client who has an open burn wound to a licensed practical nurse (LPN). Which instruction should the nurse provide to the LPN when assigning this client? a. Administer the prescribed tetanus toxoid vaccine. b. Assess the clients wounds for signs of infection. c. Encourage the client to breathe deeply every hour. d. Wash your hands on entering the clients room.

ANS: D Infection can occur when microorganisms from another person or from the environment are transferred to the client. Although all of the interventions listed can help reduce the risk for infection, handwashing is the most effective technique for preventing infection transmission.

A nurse cares for a client who has burn injuries. The clients wife asks, When will his high risk for infection decrease? How should the nurse respond? a. When the antibiotic therapy is complete. b. As soon as his albumin levels return to normal. c. Once we complete the fluid resuscitation process. d. When all of his burn wounds have closed.

ANS: D Intact skin is a major barrier to infection and other disruptions in homeostasis. No matter how much time has passed since the burn injury, the client remains at high risk for infection as long as any area of skin is open. Although the other options are important goals in the clients recovery process, they are not as important as skin closure to decrease the clients risk for infection

A nurse cares for a client who has a chest tube. When would this client be at highest risk for developing a pneumothorax? a. When the insertion site becomes red and warm to the touch b. When the tube drainage decreases and becomes sanguineous c. When the client experiences pain at the insertion sited. d. When the tube becomes disconnected from the drainage system

ANS: D Intrathoracic pressures are less than atmospheric pressures; therefore, if the chest tube becomes disconnected from the drainage system, air can be sucked into the pleural space and cause a pneumothorax. A red, warm, and painful insertion site does not increase the clients risk for a pneumothorax. Tube drainage should decrease and become serous as the client heals. Sanguineous drainage is a sign of bleeding but does not increase the clients risk for a pneumothorax.

The nurse is caring for a client who is prescribed a long-acting beta2 agonist. The client states, The medication is too expensive to use every day. I only use my inhaler when I have an attack. How should the nurse respond? a. You are using the inhaler incorrectly. This medication should be taken daily. b. If you decrease environmental stimuli, it will be okay for you to use the inhaler only for asthma attacks. c. Tell me more about your fears related to feelings of breathlessness. d. It is important to use this type of inhaler every day. Lets identify potential community services to help you.

ANS: D Long-acting beta2 agonists should be used every day to prevent asthma attacks. This medication should not be taken when an attack starts. Asthma medications can be expensive. Telling the client that he or she is using the inhaler incorrectly does not address the clients financial situation, which is the main issue here. Clients with limited incomes should be provided with community resources. Asking the client about fears related to breathlessness does not address the clients immediate concerns.

A nurse assesses a client's electrocardiograph tracing and observes that not all QRS complexes are preceded by a P wave. How would the nurse interpret this observation? a. The client has hyperkalemia causing irregular QRS complexes. b. Ventricular tachycardia is overriding the normal atrial rhythm. c. The client's chest leads are not making sufficient contact with the skin. d. Ventricular and atrial depolarizations are initiated from different sites.

ANS: D Normal rhythm shows one P wave preceding each QRS complex, indicating that all depolarization is initiated at the sinoatrial node. QRS complexes without a P wave indicate a different source of initiation of depolarization. This finding on an electrocardiograph tracing is not an indication of hyperkalemia, ventricular tachycardia, or disconnection of leads.

A nurse assesses a clients electrocardiograph tracing and observes that not all QRS complexes are preceded by a P wave. How should the nurse interpret this observation? a. The client has hyperkalemia causing irregular QRS complexes. b. Ventricular tachycardia is overriding the normal atrial rhythm. c. The clients chest leads are not making sufficient contact with the skin. d. Ventricular and atrial depolarizations are initiated from different sites.

ANS: D Normal rhythm shows one P wave preceding each QRS complex, indicating that all depolarization is initiated at the sinoatrial node. QRS complexes without a P wave indicate a different source of initiation of depolarization. This finding on an electrocardiograph tracing is not an indication of hyperkalemia, ventricular tachycardia, or disconnection of leads.

A client undergoing hemodynamic monitoring after a myocardial infarction has a right atrial pressure of 0.5 mm Hg. What action by the nurse is most appropriate? a. Level the transducer at the phlebostatic axis. b. Lay the client in the supine position. c. Prepare to administer diuretics. d. Prepare to administer a fluid bolus.

ANS: D Normal right atrial pressures are from 1 to 8 mm Hg. Lower pressures usually indicate hypovolemia, so the nurse should prepare to administer a fluid bolus. The transducer should remain leveled at the phlebostatic axis. Positioning may or may not influence readings. Diuretics would be contraindicated

A nurse cares for a client who has a pleural chest tube. Which action should the nurse take to ensure safe use of this equipment? a. Strip the tubing to minimize clot formation and ensure patency. b. Secure tubing junctions with clamps to prevent accidental disconnections. c. Connect the chest tube to wall suction at the level prescribed by the provider. d. Keep padded clamps at the bedside for use if the drainage system is interrupted.

ANS: D Padded clamps should be kept at the bedside for use if the drainage system becomes dislodged or is interrupted. The nurse should never strip the tubing. Tubing junctions should be taped, not clamped. Wall suction should be set at the level indicated by the devices manufacturer, not the provider.

A nurse is caring for a client with acute pericarditis who reports substernal precordial pain that radiates to the left side of the neck. Which nonpharmacologic comfort measure should the nurse implement? a. Apply an ice pack to the clients chest. b. Provide a neck rub, especially on the left side. c. Allow the client to lie in bed with the lights down. d. Sit the client up with a pillow to lean forward on.

ANS: D Pain from acute pericarditis may worsen when the client lays supine. The nurse should position the client in a comfortable position, which usually is upright and leaning slightly forward. Pain is decreased by using gravity to take pressure off the heart muscle. An ice pack and neck rub will not relieve this pain.

A client is in the preoperative holding area prior to an emergency coronary artery bypass graft (CABG). The client is yelling at family members and tells the doctor to just get this over with when asked to sign the consent form. What action by the nurse is best? a. Ask the family members to wait in the waiting area. b. Inform the client that this behavior is unacceptable. c. Stay out of the room to decrease the clients stress levels. d. Tell the client that anxiety is common and that you can help.

ANS: D Preoperative fear and anxiety are common prior to cardiac surgery, especially in emergent situations. The client is exhibiting anxiety, and the nurse should reassure the client that fear is common and offer to help. The other actions will not reduce the clients anxiety.

A nurse assesses a client who has burn injuries and notes crackles in bilateral lung bases, a respiratory rate of 40 breaths/min, and a productive cough with blood-tinged sputum. Which action should the nurse take next? a. Administer furosemide (Lasix). b. Perform chest physiotherapy. c. Document and reassess in an hour. d. Place the client in an upright position.

ANS: D Pulmonary edema can result from fluid resuscitation given for burn treatment. This can occur even in a young healthy person. Placing the client in an upright position can relieve lung congestion immediately before other measures can be carried out. Although Lasix may be used to treat pulmonary edema in clients who are fluid overloaded, a client with a burn injury will lose a significant amount of fluid through the broken skin; therefore, Lasix would not be appropriate. Chest physiotherapy will not get rid of fluid.

A student nurse asks for an explanation of refractory hypoxemia. What answer by the nurse instructor is best? A. It is chronic hypoxemia that accompanies restrictive airway disease. B. It is hypoxemia from lung damage due to mechanical ventilation. C. It is hypoxemia that continues even after the client is weaned from oxygen. D. It is hypoxemia that persists even with 100% oxygen administration.

ANS: D Refractory hypoxemia is hypoxemia that persists even with the administration of 100% oxygen. It is a cardinal sign of acute respiratory distress syndrome. It does not accompany restrictive airway disease and is not caused by the use of mechanical ventilation or by being weaned from oxygen.

A client is on mechanical ventilation and the clients spouse wonders why ranitidine (Zantac) is needed since the client only has lung problems. What response by the nurse is best? A. It will increase the motility of the gastrointestinal tract. B. It will keep the gastrointestinal tract functioning normally. C. It will prepare the gastrointestinal tract for enteral feedings. D. It will prevent ulcers from the stress of mechanical ventilation.

ANS: D Stress ulcers occur in many clients who are receiving mechanical ventilation, and often prophylactic medications are used to prevent them. Frequently used medications include antacids, histamine blockers, and proton pump inhibitors. Zantac is a histamine blocking agent.

A nurse assesses several clients who have a history of asthma. Which client should the nurse assess first? a. A 66-year-old client with a barrel chest and clubbed fingernails b. A 48-year-old client with an oxygen saturation level of 92% at rest c. A 35-year-old client who has a longer expiratory phase than inspiratory phase d. A 27-year-old client with a heart rate of 120 beats/min

ANS: D Tachycardia can indicate hypoxemia as the body tries to circulate the oxygen that is available. A barrel chest is not an emergency finding. Likewise, a pulse oximetry level of 92% is not considered an acute finding. The expiratory phase is expected to be longer than the inspiratory phase in someone with airflow limitation.

A nurse cares for a client who has advanced cardiac disease and states, I am having trouble sleeping at night. How should the nurse respond? a. I will consult the provider to prescribe a sleep study to determine the problem. b. You become hypoxic while sleeping; oxygen therapy via nasal cannula will help. c. A continuous positive airway pressure, or CPAP, breathing mask will help you breathe at night. d. Use pillows to elevate your head and chest while you are sleeping.

ANS: D The client is experiencing orthopnea (shortness of breath while lying flat). The nurse should teach the client to elevate the head and chest with pillows or sleep in a recliner. A sleep study is not necessary to diagnose this client. Oxygen and CPAP will not help a client with orthopnea.

A nurse cares for a client with end-stage heart failure who is awaiting a transplant. The client appears depressed and states, I know a transplant is my last chance, but I dont want to become a vegetable. How should the nurse respond? a. Would you like to speak with a priest or chaplain? b. I will arrange for a psychiatrist to speak with you. c. Do you want to come off the transplant list? d. Would you like information about advance directives?

ANS: D The client is verbalizing a real concern or fear about negative outcomes of the surgery. This anxiety itself can have a negative effect on the outcome of the surgery because of sympathetic stimulation. The best action is to allow the client to verbalize the concern and work toward a positive outcome without making the client feel as though he or she is crazy. The client needs to feel that he or she has some control over the future. The nurse personally provides care to address the clients concerns instead of pushing the clients issues off on a chaplain or psychiatrist. The nurse should not jump to conclusions and suggest taking the client off the transplant list, which is the best treatment option.

A nurse is assessing a client who had a myocardial infarction. Upon auscultating heart sounds, the nurse hears the following sound. What action by the nurse is most appropriate?(Click the media button to hear the audio clip.) a. Assess for further chest pain. b. Call the Rapid Response Team. c. Have the client sit upright. d. Listen to the clients lung sounds.

ANS: D The sound the nurse hears is an S3 heart sound, an abnormal sound that may indicate heart failure. The nurse should next assess the clients lung sounds. Assessing for chest pain is not directly related. There is no indication that the Rapid Response Team is needed. Having the client sit up will not change the heart sound.

A nurse cares for a client who has packing inserted for posterior nasal bleeding. Which action should the nurse take first? a. Assess the clients pain level. b. Keep the clients head elevated. c. Teach the client about the causes of nasal bleeding. d. Make sure the string is taped to the clients cheek.

ANS: D The string should be attached to the clients cheek to hold the packing in place. The nurse needs to make sure that this does not move because it can occlude the clients airway. The other options are good interventions, but ensuring that the airway is patent is the priority objective.

A nurse is caring for a client on mechanical ventilation. When double-checking the ventilator settings with the respiratory therapist, what should the nurse ensure as a priority? A. The client is able to initiate spontaneous breaths. B. The inspired oxygen has adequate humidification. C. The upper peak airway pressure limit alarm is off. D. The upper peak airway pressure limit alarm is on.

ANS: D The upper peak airway pressure limit alarm will sound when the airway pressure reaches a preset maximum. This is critical to prevent damage to the lungs. Alarms should never be turned off. Initiating spontaneous breathing is important for some modes of ventilation but not others. Adequate humidification is important but does not take priority over preventing injury.

A nurse assesses a client after administering isosorbide mononitrate (Imdur). The client reports a headache. Which action should the nurse take? a. Initiate oxygen therapy. b. Hold the next dose of Imdur. c. Instruct the client to drink water. d. Administer PRN acetaminophen.

ANS: D The vasodilating effects of isosorbide mononitrate frequently cause clients to have headaches during the initial period of therapy. Clients should be told about this side effect and encouraged to take the medication with food. Some clients obtain relief with mild analgesics, such as acetaminophen. The clients headache is not related to hypoxia or dehydration; therefore, these interventions would not help. The client needs to take the medication as prescribed to prevent angina; the medication should not be held.

A client is brought to the emergency department after sustaining injuries in a severe car crash. The clients chest wall does not appear to be moving normally with respirations, oxygen saturation is 82%, and the client is cyanotic. What action by the nurse is the priority? A. Administer oxygen and reassess. B. Auscultate the clients lung sounds. C. Facilitate a portable chest x-ray. D. Prepare to assist with intubation.

ANS: D This client has manifestations of flail chest and, with the other signs, needs to be intubated and mechanically ventilated immediately. The nurse does not have time to administer oxygen and wait to reassess, or to listen to lung sounds. A chest x-ray will be taken after the client is intubated.

A nurse prepares to defibrillate a client who is in ventricular fibrillation. Which intervention is appropriate for the nurse to perform prior to defibrillating this client? a. Make sure that the defibrillator is set to the synchronous mode. b. Administer 1 mg of intravenous epinephrine. c. Test the equipment by delivering a smaller shock at 100 J. d. Ensure that everyone is clear of contact with the client and the bed.

ANS: D To avoid injury, the rescuer commands that all personnel clear contact with the client or the bed and ensures their compliance before delivery of the shock. Defibrillation is done in asynchronous mode. Equipment would not be tested before a client is defibrillated because this is an emergency procedure; equipment would be checked on a routine basis. Defibrillation takes priority over any medications.

A nurse prepares to defibrillate a client who is in ventricular fibrillation. Which priority intervention should the nurse perform prior to defibrillating this client? a. Make sure the defibrillator is set to the synchronous mode. b. Administer 1 mg of intravenous epinephrine. c. Test the equipment by delivering a smaller shock at 100 joules. d. Ensure that everyone is clear of contact with the client and the bed.

ANS: D To avoid injury, the rescuer commands that all personnel clear contact with the client or the bed and ensures their compliance before delivery of the shock. A precordial thump can be delivered when no defibrillator is available. Defibrillation is done in asynchronous mode. Equipment should not be tested before a client is defibrillated because this is an emergency procedure; equipment should be checked on a routine basis. Epinephrine should be administered after defibrillation.


Set pelajaran terkait

ITIL 4 - Create, Support, and Deliver

View Set

Ch. 16 Monopoly Practice Problems

View Set